SlideShare a Scribd company logo
1 of 68
Download to read offline
A 3-month-old boy presents to your outpatient clinic as the parents describe seeing clear fluid
discharge from the umbilicus. This has been noted since soon after the umbilical cord separated.
Which one of the following is the most likely underlying (embryological) explanation?
Select one:
A patent urachus
A patent vitello-intestinal duct
A urachal cyst
An exomphalos
An umbilical granuloma
Certainty: C=1 (Unsure: <67%) C=2 (Mid: >67%) C=3 (Quite sure: >80%)
Feedback
After separation of the cord there should not be any discharge from the umbilicus. The presence of
clear fluid points towards this being urine rather than bowel content coming from the umbilicus. A
patent urachus implies a persistent collection to the bladder, allowing small amounts of urine to leak
but the patient often remains completely well.
A patent urachus - correct
A patent vitello-intestinal duct - you would expect bowel content, not clear fluid with a connection to
the bowel
A urachal cyst - a urachal cyst typically will not drain as the urachal tract is obliterated on either side
of the cyst
An exomphalos - this is an abdominal wall defect with the bowel covered by the cord structures and
would be identified soon after birth
An umbilical granuloma - a granuloma may leak small amounts of fluid but typically is more
serosanguinous
The correct answer is:
A patent urachus
You did not select a certainty. Assuming: C=1 (Unsure: <67%).
Question 2
Incorrect
CBM mark 0.00
Weight 1.00
Flag question
Question text
A 40-year-old man presents with clinical and endocrinological features of acromegaly. A
microadenoma of the pituitary gland is identified. Which one of the following is the most compelling
reason for treating this tumour?
Select one:
Avoidance of cardiovascular complications
Avoidance of colonic cancer
Avoidance of diabetes
Avoidance of optic pathway compression
Avoidance of radiological progression
Certainty: C=1 (Unsure: <67%) C=2 (Mid: >67%) C=3 (Quite sure: >80%)
Feedback
Cardiovascular complications have most effect on survival and are best avoided by early treatment.
All other conditions can occur but have less effect upon survival and are therefore less compelling.
Radiological progression would initiate additional treatment. Optic pathway compression will not
occur with a micro adenoma unless significant growth occurs.
The correct answer is:
Avoidance of cardiovascular complications
You did not select a certainty. Assuming: C=1 (Unsure: <67%).
Question 3
Correct
CBM mark 1.00
Weight 1.00
Flag question
Question text
Whilst on call for orthopaedics you diagnose a child with septic arthritis and you are to assist your
consultant in theatre performing a washout of the hip joint.
The approach used is the anterior approach between the rectus femoris and gluteus medius
muscles. Which one of the following inter-nervous planes is used in this approach?
Select one:
Direct rami of the lumbar plexus and the inferior gluteal nerve
Femoral nerve and superior gluteal nerve
Lateral cutaneous nerve and inferior gluteal nerve
Lateral cutaneous nerve and super gluteal nerve
Super gluteal nerve and inferior gluteal nerve
Certainty: C=1 (Unsure: <67%) C=2 (Mid: >67%) C=3 (Quite sure: >80%)
Feedback
The anterior approach is commonly used in paediatrics and exploits the inter-nervous interval
between femoral nerve (sartorius and rectus femoris) and superior gluteal nerve ( tensor fascia latae
and gluteus medius).
The correct answer is:
Femoral nerve and superior gluteal nerve
You did not select a certainty. Assuming: C=1 (Unsure: <67%).
Question 4
Correct
CBM mark 1.00
Weight 1.00
Flag question
Question text
A 1-year-old girl presents with shortness of breath. Her parents have also noticed that her eyes no
longer look symmetrical. The right pupil is smaller than the left and the right eyelid appears droopy.
A chest X-ray shows increased opacification in the right hemithorax, particularly in the right upper
zone. Which one of the following is the most likely diagnosis?
Select one:
A community acquired pneumonia
A congenital diaphragmatic hernia
A congenital pulmonary airway malformations
Lymphoma
Neuroblastoma
Certainty: C=1 (Unsure: <67%) C=2 (Mid: >67%) C=3 (Quite sure: >80%)
Feedback
The clinical features point to a right sided Horner syndrome, secondary to a right apical tumour
compressing the sympathetic chain. In this age group the most likely cause is neuroblastoma.
A community acquired pneumonia - although a pneumonia can present with shortness of breath, the
Horner syndrome is not in keeping
A congenital diaphragmatic hernia - although late presenting congenital diaphragmatic hernia can
present with shortness of breath, the Horner syndrome is not in keeping
A congenital pulmonary airway malformations - although late presenting congenital pulmonary
airway malformation can present with shortness of breath (usually because of infection in the
CPAM), the Horner syndrome is not in keeping
Lymphoma - lymphoma causes lymph node enlargement. The chest X-ray may typically show a
mediastinal mass
Neuroblastoma - correct
The correct answer is:
Neuroblastoma
You did not select a certainty. Assuming: C=1 (Unsure: <67%).
Question 5
Correct
CBM mark 1.00
Weight 1.00
Flag question
Question text
A 25-year-old woman presents with sensory symptoms in the upper limbs and a headache that is
worse on coughing. An MRI scan shows a Chiari malformation of the hindbrain and syringomyelia in
the cervical spine. Which one of the following nerve tracts are most vulnerable to damage in this
condition?
Select one:
Corticospinal
Posterior columns
Spinocerebellar
Spinothalamic
Sympathetic trunks
Certainty: C=1 (Unsure: <67%) C=2 (Mid: >67%) C=3 (Quite sure: >80%)
Feedback
A syrinx expands the centre of the spinal cord. The crossing fibres of the spinothalamic are most at
risk of injury. All other fibres are further from the site of pathology.
The correct answer is:
Spinothalamic
You did not select a certainty. Assuming: C=1 (Unsure: <67%).
Question 6
Incorrect
CBM mark 0.00
Weight 1.00
Flag question
Question text
An elderly patient with an intra-capsular fractured neck of femur is due to have a hip
hemiarthroplasty performed. Which of the following describe anatomical steps to access the hip joint
during the most commonly used approach for a hemiarthroplasty?
Select one:
Detaching the anterior aspect of gluteus medius and minimus from the greater trochanter
Detaching piriformis and the short external rotators from their femoral insertion
Developing a plane between the gracilis and adductor longus muscles
Developing a superficial plane between sartorius and tensor fasciae latae
Elevation of vastus lateralis
Certainty: C=1 (Unsure: <67%) C=2 (Mid: >67%) C=3 (Quite sure: >80%)
Feedback
The most commonly used approach for hip hemiarthroplasty is the antero - lateral (modified
hardinge) approach to the hip. During this approach the gluteus medius and minimus are detached
as one or separate layers from the greater trochanter anteriorly to gain access to the hip joint.
The correct answer is:
Detaching the anterior aspect of gluteus medius and minimus from the greater trochanter
You did not select a certainty. Assuming: C=1 (Unsure: <67%).
Question 7
Correct
CBM mark 1.00
Weight 1.00
Flag question
Question text
An 80-year-old woman is admitted with a pulsatile mass in the abdomen, guarding and tenderness.
An ultrasound demonstrates an intact abdominal aortic aneurysm at the level of the origin of the
superior mesenteric artery. You suspect bowel ischaemia of which of the following structures?
Select one:
The jejunum, ileum and ascending colon
The kidneys and spleen
The liver and pancreas
The stomach and duodenum
The transverse and descending colon
Certainty: C=1 (Unsure: <67%) C=2 (Mid: >67%) C=3 (Quite sure: >80%)
Feedback
The celiac trunk supplies the foregut (the liver, stomach and spleen); the SMA supplies the midgut
(small intestine and ascending colon) and the IMA supplies the hindgut (transverse, descending and
sigmoid colon).
The correct answer is:
The jejunum, ileum and ascending colon
You did not select a certainty. Assuming: C=1 (Unsure: <67%).
Question 8
Correct
CBM mark 1.00
Weight 1.00
Flag question
Question text
A 73-year-old man is admitted with severe abdominal pain and guarding. He also has painful tips of
the radial 2 digits which are dusky. He is haemodynamically stable but has untreated atrial
fibrillation. You suspect an underlying arterioembolic cause and organise an urgent CT angiogram of
the bowel, which shows that the inferior mesenteric artery is occluded. Which one of the following
will result from the occluded artery?
Select one:
Ischaemia of the descending colon & sigmoid colon
Ischaemia of the duodenum
Ischaemia of the jejunum
Ischaemia of the jejunum and ileum
Ischaemia of the terminal ileum and ascending colon
Certainty: C=1 (Unsure: <67%) C=2 (Mid: >67%) C=3 (Quite sure: >80%)
Feedback
This patient is is suffering with arterial emboli which has led to ischaemia in both the upper limb and
bowel.
The inferior mesenteric artery supplies the left side of the colon and sigmoid and branches include:
left colic
ascending branch that joins the middle colic
descending branch that joins the highest sigmoid branch
sigmoid arteries (2-3)
superior sigmoid branch join the left colic
inferior sigmoid branch joins the superior rectal
The superior mesenteric artery supplies the small bowel and right side of the colon.
The correct answer is:
Ischaemia of the descending colon & sigmoid colon
You did not select a certainty. Assuming: C=1 (Unsure: <67%).
Question 9
Correct
CBM mark 1.00
Weight 1.00
Flag question
Question text
A patient who was involved in a road traffic accident suffers a penetrating trauma to the infra-
clavicular area and is found to have a pulsatile mass at the level of pectorals minor. You suspect an
injury to the axillary artery and the lateral cord of the brachial plexus due to loss of function of which
of the following nerves?
Select one:
Axillary and median nerves
Median and musculocutaneous nerves
Musculocutaneous and radial nerves
Posterior interosseous and anteriorinterosseous nerves
Ulnar and median nerves
Certainty: C=1 (Unsure: <67%) C=2 (Mid: >67%) C=3 (Quite sure: >80%)
Feedback
The median and musculocutaneous nerves both arise from the lateral cord of the brachial plexus.
The axillary nerve arises from the posterior cord, as does the posterior interosseous branch of the
radial nerve. The anterior interosseous is a branch of the median nerve.
The correct answer is:
Median and musculocutaneous nerves
You did not select a certainty. Assuming: C=1 (Unsure: <67%).
Question 10
Incorrect
CBM mark 0.00
Weight 1.00
Flag question
Question text
A 9-year-old boy presents with a non-retractile foreskin. He does not currently have any urinary
symptoms. On examination the foreskin is thickened, white and stiff. Which one of the following is
the most appropriate next step?
Select one:
A circumcision
A preputioplasty
Oral steroids
Topical betnovate cream
Waiting until post puberty to reassess
Certainty: C=1 (Unsure: <67%) C=2 (Mid: >67%) C=3 (Quite sure: >80%)
Feedback
The history points to balanitis xerotica obliterans. This is a progressive scarring condition and
requires a circumcision for definitive treatment.
A - correct
B - this loosens the foreskin but does not remove the disease process
C - oral steroids have no place in the management of this localised disease
D - Betnovate may control the symptoms for a short period but is not definitive.
E - there is no advantage in waiting and there is a small risk of urinary symptoms progressing to the
point of urinary retention
The correct answer is:
A circumcision
You did not select a certainty. Assuming: C=1 (Unsure: <67%).
The morning following a successful right carotid endarterectomy under general anaesthesia, a 65-
year-old man complains of severe hoarseness. Which one of the following is the most likely
diagnosis?
Select one:
The hypoglossal nerve has been damaged during surgery
The larynx has been traumatised during anaesthesia
The patient has sustained a post operative stroke
The recurrent laryngeal nerve has been damaged during surgery
There has been bleeding from the endarterectomy site
Certainty: C=1 (Unsure: <67%) C=2 (Mid: >67%) C=3 (Quite sure: >80%)
Feedback
A number of cranial nerves are at risk during carotid surgery and hoarseness secondary to recurrent
laryngeal nerve palsy is one of the more common complications. The recurrent laryngeal nerve is at
risk during exposure and endarterectomy of the carotid bifurcation due to proximity of the vagus
nerve.
The correct answer is:
The recurrent laryngeal nerve has been damaged during surgery
You did not select a certainty. Assuming: C=1 (Unsure: <67%).
Question 12
Correct
CBM mark 1.00
Weight 1.00
Flag question
Question text
A 10-year-old boy presents to the Emergency Department. Last week he had an upper respiratory
tract infection, following which he quickly developed swelling around his right eye and was unable to
open it. Now he has bilateral periorbital swelling, a temperature of 40oC and a Glasgow Coma Scale
score of 13. He has a contrast enhanced CT scan which demonstrates a cavernous sinus
thrombosis. Which one of the following symptoms/signs would not be present?
Select one:
Absent corneal reflex
Anaesthesia over the angle of the mandible
Ophthalmoplegia
Lateral rectus palsy
Absent swinging light pupillary reflex
Certainty: C=1 (Unsure: <67%) C=2 (Mid: >67%) C=3 (Quite sure: >80%)
Feedback
Infected thrombus within the cavernous sinus may cause a variety of cranial nerve palsies affecting
those nerves that are directly related to the sinus.
Cranial nerves III, IV, V1 and V2 run within the lateral wall of the cavernous sinus.
Lesions of CN III in isolation causes the globe to rest in downward, lateral gaze, and interruption of
the efferent limb of the pupillary reflexes to light (tested by the swinging light reflex) and
accommodation.
Lesions of CN IV causes diplopia in downward gaze doe to paralysis of Superior Oblique muscle;
while lesions of CN VI cause paralysis of lateral gaze (Lateral Rectus muscle). A combined lesion of
CN III, IV and VI causes complete ophthalmoplegia.
CN V1, the ophthalmic nerve supplies sensation to the upper face, skull and eye, so a lesion of it
affects the afferent limb of the corneal reflex.
Sensation to the angle of the mandible is supplied by the mandibular nerve (CN V3), which is not
directly related to the cavernous sinus.
The correct answer is:
Anaesthesia over the angle of the mandible
You did not select a certainty. Assuming: C=1 (Unsure: <67%).
Question 13
Correct
CBM mark 1.00
Weight 1.00
Flag question
Question text
A 50-year-old man presents with a history of severe low back pain referred down the right leg to the
foot. He reports saddle numbness and difficulty with micturition having not had any sensation to pass
urine for more than 12 hours. On examination, straight leg raising is limited to 30 degrees on the
right and 60 degrees on the left. Motor assessment is difficult due to pain but there appears to be
weakness of ankle plantar flexion and eversion. Pinprick and light touch sensation are reduced in the
S1 - S4 dermatomes bilaterally. An MRI scan is performed. Which one of the following is the most
likely diagnosis?
Select one:
Discitis
Epidural haematoma
Prolapsed intervertebral disc
S1 root neurofibroma
Vertebral haemangioma
Certainty: C=1 (Unsure: <67%) C=2 (Mid: >67%) C=3 (Quite sure: >80%)
Feedback
All pathologies listed occur in this region. The disc has obviously prolapsed on the scan - this fits
with the clinical history.
The correct answer is:
Prolapsed intervertebral disc
You did not select a certainty. Assuming: C=1 (Unsure: <67%).
Question 14
Correct
CBM mark 1.00
Weight 1.00
Flag question
Question text
A 2-week-old baby has been unable to breast feed since birth and is struggling to bottle feed. The
baby cries persistently and according to mum seems to 'burp' and suffer trapped wind. On bottle
feeding mother reports regurgitation of milk through the nose. The baby appears underweight and
you are concerned about failure to thrive. Which one of the following is the most likely diagnosis?
Select one:
Branchial cyst
Cleft palate
Hiatus hernia
Pharyngeal pouch
Pyloric stenosis
Certainty: C=1 (Unsure: <67%) C=2 (Mid: >67%) C=3 (Quite sure: >80%)
Feedback
Cleft palate causes difficulty in making an adequate seal/suction for breast feeding.
Due to excessive air intake bottle feeding is also difficult and results in trapped wind and bloating
and milk regurgitation through the nose.
Hiatus hernia causes reflux and or vomiting but does not present with breast or bottle feeding issues.
Branchial cysts usually present with a midline swelling.
Pyloric stenosis presents with projectile vomiting.
Pharyngeal pouches present in later life.
The correct answer is:
Cleft palate
You did not select a certainty. Assuming: C=1 (Unsure: <67%).
Question 15
Incorrect
CBM mark 0.00
Weight 1.00
Flag question
Question text
A 65-year-old woman develops a hoarse voice, evident in theatre recovery, following an anterior
cervical discectomy at the C3/4 level. Anaesthesia was uncomplicated. Surgery, via a 4 cm
transverse skin crease incision, was uneventful. A post-operative MRI scan shows no evidence of a
haematoma. Which one of the following is the most likely explanation for her symptoms?
Select one:
External laryngeal nerve transection
Laryngeal oedema
Recurrent laryngeal nerve transection
Superior laryngeal nerve neuropraxis
Vagal nerve neuropraxis
Certainty: C=1 (Unsure: <67%) C=2 (Mid: >67%) C=3 (Quite sure: >80%)
Feedback
Recurrent larnygeal not at risk for surgery at this level (much more common with C6/7 pathology).
Laryngeal oedema could be a cause but given anaesthetic uncomplicated unlikely. The superior
laryngeal is at risk in upper cervical surgery - more commonly due to retraction (neuropraxis) than
transection.
The correct answer is:
Superior laryngeal nerve neuropraxis
You did not select a certainty. Assuming: C=1 (Unsure: <67%).
Question 16
Correct
CBM mark 1.00
Weight 1.00
Flag question
Question text
You see a 56-year-old man in clinic who has presented with left submandibular swelling related to
eating. He has had an ultrasound scan which shows a 1.5 cm stone within the substance of the left
submandibular gland and you discuss the management options with him, including surgical removal
of the gland. Which of the following nerves would you tell him are at risk during this operation?
Select one:
Accessory, marginal mandibular and lingual nerves
Accessory, hypoglossal and lingual nerves
Glossopharyngeal, marginal mandibular and lingual nerves
Hypoglossal, marginal mandibular and lingual nerves
Marginal mandibular, hypoglossal and glossopharyngeal nerves
Certainty: C=1 (Unsure: <67%) C=2 (Mid: >67%) C=3 (Quite sure: >80%)
Feedback
Damage to the hypoglossal nerve causes deviation of the tongue to the affected side. Damage to
the marginal mandibular nerve causes weakness of the depressor anguli oris muscle, which pulls
the corner of the lower lip downwards. Damage to the lingual nerve causes numbness of the
ipsilateral side of the tongue.
The correct answer is:
Hypoglossal, marginal mandibular and lingual nerves
You did not select a certainty. Assuming: C=1 (Unsure: <67%).
Question 17
Incorrect
CBM mark 0.00
Weight 1.00
Flag question
Question text
A 32-year-old patient presents to the Emergency Department following a stab injury to the neck. You
note a small wound in the anterior triangle and a small pulsatile swelling approximately at the level of
the hyoid bone. You also suspect a nerve injury. Which one of the following indicates this?
Select one:
Failure to 'screw up the eyes' tightly
Hearing loss
Inability to tense the masseter or temporals muscles
Loss of sensation to the upper lip
Loss of taste to the anterior 2/3rds of the tongue
Certainty: C=1 (Unsure: <67%) C=2 (Mid: >67%) C=3 (Quite sure: >80%)
Feedback
Many cranial nerves transit the anterior triangle of the neck, namely VII, IX, X, XI and XII. A failure to
screw up the eyes indicates an injury to the facial nerve.
The other responses indicate damage to nerves that do not transit the anterior triangle; hearing loss
would indicate injury to VIII, the anterior 2/3rds of the tongue is supplied by the trigeminal nerve as is
sensation to the lip and motor innervation of the masseter and temporals.
The correct answer is:
Failure to 'screw up the eyes' tightly
You did not select a certainty. Assuming: C=1 (Unsure: <67%).
Question 18
Incorrect
CBM mark 0.00
Weight 1.00
Flag question
Question text
A pregnant 35-year-old woman is being advised of some potential risks of her new-born child
suffering developmental dysplasia of the hip (DDH). Which one of the following is the most
significant risk factor associated with DDH?
Select one:
Breech presentation
Family history
Female child
Firstborn
Oligohydramnios
Certainty: C=1 (Unsure: <67%) C=2 (Mid: >67%) C=3 (Quite sure: >80%)
Feedback
The main risk associated with DDH is breech presentation. Female babies are at higher risk than
males (ratio of 6:1 female to male). Family history and oligohydramnios are also contributing factors.
The correct answer is:
Breech presentation
You did not select a certainty. Assuming: C=1 (Unsure: <67%).
Question 19
Incorrect
CBM mark 0.00
Weight 1.00
Flag question
Question text
A 46-year-old woman presents to the Emergency Department with a headache, diplopia and ocular
pain, two weeks following upper respiratory tract infection. She is an insulin dependant diabetic who
suffers with stage 2 chronic kidney disease and mild hypertension. Her blood pressure is 130/88, her
HBA1C is 84 and her blood glucose levels in the last 14 days have been averaging 16-18mmol/l.
Her pulse is 80bpm and regular. Her right eye is pointing slightly down and outwards. Which of the
following explains her symptoms best?
Select one:
Cerebrovascular accident
Hemiplegic migraine
Intra-cranial hypertension
Mono-neuritis
Temporal arteritis
Certainty: C=1 (Unsure: <67%) C=2 (Mid: >67%) C=3 (Quite sure: >80%)
Feedback
This is a complicated history with a number of associated features. The finding of note is the
downwards and outwards facing eye; indicative of a 3rd cranial nerve (oculomotor) palsy. The
aetiology of oculomotor palsy is must-factorial, however, in this history, the patient is not significantly
hypertensive and has a regular pulse; unlikely to be a CVA (bleed or embolic). She is however
diabetic with significantly raised blood glucose over a number of months (HBA1C). Diabetes is a
major cause of mono-neuritis.
The correct answer is:
Mono-neuritis
You did not select a certainty. Assuming: C=1 (Unsure: <67%).
Question 20
Incorrect
CBM mark 0.00
Weight 1.00
Flag question
Question text
A 32-year-old man presents to the Emergency Department having come off his mountain bike and
hit a tree. He undergoes primary and secondary surveys and the only abnormalities found are
bruising over his right mastoid process (Battle's sign) and a complete lower motor neurone facial
paralysis. You suspect a temporal bone fracture. Which one of the following examinations or
investigations would not be helpful in identifying the level of his facial nerve injury?
Select one:
Hearing test
Pupillary light reflex
Salivary flow rate
Schirmer's test of lacrimation
Stapedial reflex testing
Certainty: C=1 (Unsure: <67%) C=2 (Mid: >67%) C=3 (Quite sure: >80%)
Feedback
Bruising over the mastoid process (Battle's sign) is a sign of a temporal bone fracture. The facial
nerve (CN VII) exits the brainstem at the cerebellopontine angle and runs through the temporal
bone, initially in close proximity to the vestibulocochlear nerve (CN VIII) in the internal acoustic
meatus, and gives off the following branches during its intratemporal course: greater superficial
petrosal nerve (parasympathetic fibres to the lacrimal, nasal and palatine glands via the
pterygopalatine ganglion), nerve to stapedius, chorda tympani (carrying taste sensation from the
anterior 2/3 of the tongue and parasympathetic fibres to the submandibular ganglion). Pupillary light
reflex involves the optic (CN II) and oculomotor (CN III) nerves for its afferent and efferent limbs.
The correct answer is:
Pupillary light reflex
You did not select a certainty. Assuming: C=1 (Unsure: <67%).
A 5-year-old boy presents to the Emergency Department with a short history of an upper respiratory
tract infection that has developed into a left neck swelling, neck stiffness and trismus. He is admitted
and treated with intravenous co-amoxiclav, but his symptoms have not improved after 24 hours so
you decide to arrange an MRI to exclude a deep neck space infection. The scan shows a collection
posterior to the retropharyngeal space, between the alar and prevertebral fascia, and extending from
the skull base into the chest. Which one of the following is the lower limit of this space?
Select one:
It fuses with the diaphragm
It fuses with the pericardium
T1-2 vertebral level
T3-4 vertebral level
T5-6 vertebral level
Certainty: C=1 (Unsure: <67%) C=2 (Mid: >67%) C=3 (Quite sure: >80%)
Feedback
The deep spaces of the neck are potential routes for spread of infection. The prevertebral fascia
anteriorly splits into the alar and prevertebral fascial layers. The potential space between them
contains loose connective tissue and extends from the skull base to the diaphragm. It is often called
the 'danger space' since it provides a pathway for infections of the head and neck to spread into the
chest, causing mediastinitis and mediastinal abscess.
Other tissue spaces in the neck include: Carotid space (extends from skull base to aortic arch);
retropharyngeal space (extends from skull base to T1/2); visceral space (extends from hyoid bone
and opens into superior mediastinum).
The correct answer is:
It fuses with the diaphragm
You did not select a certainty. Assuming: C=1 (Unsure: <67%).
Question 22
Incorrect
CBM mark 0.00
Weight 1.00
Flag question
Question text
A 41-year-old man is admitted as an emergency to the plastic surgery ward with cellulitis of the
perineal area. It rapidly spreads, resulting in Fournier's gangrene (necrotising fasciitis) and
widespread sepsis. This necessitates multiple trips to theatre and admission to ITU. Where is the
infection likely to have originated?
Select one:
Camper fascia
Corpora cavernosa and testes
External and internal spermatic fascia
Scarpa fascia and Colles fascia
The inferior fascia of the urogenital diaphragm and Colles fascia
Certainty: C=1 (Unsure: <67%) C=2 (Mid: >67%) C=3 (Quite sure: >80%)
Feedback
The perineal membrane (also known as the inferior fascia of the urogenital diaphragm) and Colles
fascia, define an area known as the superficial perineal space.
The superficial perineal space contains the membraneous and bulbar urethra and bulbourethral
glands. It is adjacent to the anterior anal wall and ischiorectal fossae. Infectious disease of the
urethra, bulbourethral glands, perineal structures, or rectum can drain into the superficial perineal
space. It may spread into the scrotum or into the anterior abdominal wall and rapidly cephalad via
the fascial planes as far as the level of the clavicles.
The correct answer is:
The inferior fascia of the urogenital diaphragm and Colles fascia
You did not select a certainty. Assuming: C=1 (Unsure: <67%).
Question 23
Correct
CBM mark 1.00
Weight 1.00
Flag question
Question text
An 80-year-old man who normally walks short distances with a frame and has a past medical history
of angina and diabetes, falls and sustains a displaced intracapsular hip fracture. Which of the
following is the most appropriate definitive management for him?
Select one:
Bed rest and traction applied to the injured leg
Surgery to perform a dynamic hip screw
Surgery to perform a hip hemiarthroplasty
Surgery to perform a total hip replacement
Surgery to perform open reduction and internal fixation with cannulated screws
Certainty: C=1 (Unsure: <67%) C=2 (Mid: >67%) C=3 (Quite sure: >80%)
Feedback
This is an intracapsular fracture and therefore the blood supply to the head is deemed to be
damaged and therefore the most appropriate treatment is to removed the head and perform a
hemiarthroplasty. This allows the patient to be mobilised and reduces analgesic requirements. A
dynamic hip screw is appropriate for extra capsular fractures where the blood supply to the head is
presumed intact. He would not be suitable for a total hip replacement due to his co morbidities and
limited mobilisation. Cannulated screws are performed when trying to preserve the femoral head
such as in young patients or minimally displaced intracapsular fractures where the blood supply is
more likely to remain intact. They my also require the patient to be less than full weight bearing and
so are not ideal in elderly people who require frames to walk. Bed rest and traction is reserved for
the very few numbers of patients who remain unfit for an anaesthetic despite optimisation.
The correct answer is:
Surgery to perform a hip hemiarthroplasty
You did not select a certainty. Assuming: C=1 (Unsure: <67%).
Question 24
Correct
CBM mark 1.00
Weight 1.00
Flag question
Question text
Deviation of the tongue to the right, on attempted protrusion of the tongue, would indicate
damage/dysfunction in which of the following cranial nerves?
Select one:
Left cranial accessory nerve
Left hypoglossal nerve
Right glossopharyngeal nerve
Right hypoglossal nerve
Right lingual nerve
Certainty: C=1 (Unsure: <67%) C=2 (Mid: >67%) C=3 (Quite sure: >80%)
Feedback
Each hypoglossal nerve innervates all the extrinsic and intrinsic lingual muscles ipsilaterally except
the palatoglossus muscle, the latter being innervated by the vagus via the pharyngeal plexus. The
cranial accessory and glossopharyngeal nerves do not innervate any of the lingual muscles and the
lingual nerve does not supply any muscles.
Deviation of the tongue to the right would indicate paralysis of the right genioglossus muscle and
unopposed action of the left genioglossus.
The correct answer is:
Right hypoglossal nerve
You did not select a certainty. Assuming: C=1 (Unsure: <67%).
Question 25
Correct
CBM mark 1.00
Weight 1.00
Flag question
Question text
Which extraocular orbital muscle is supplied by the trochlear nerve (IV th cranial nerve)?
Select one:
Inferior oblique
Inferior rectus
Medial rectus
Superior oblique
Superior rectus
Certainty: C=1 (Unsure: <67%) C=2 (Mid: >67%) C=3 (Quite sure: >80%)
Feedback
The superior rectus is supplied by the superior division of the oculomotor nerve while the medial
rectus, inferior rectus and inferior oblique are supplied by the inferior division of the oculomotor
nerve.
The correct answer is:
Superior oblique
You did not select a certainty. Assuming: C=1 (Unsure: <67%).
Question 26
Correct
CBM mark 1.00
Weight 1.00
Flag question
Question text
The superficial 'lobe' of the submandibular gland is separated from the deep 'lobe' by which of the
following muscles?
Select one:
Hyoglossus
Mylohyoid
Geniohyoid
Stylohyoid
Genioglossus
Certainty: C=1 (Unsure: <67%) C=2 (Mid: >67%) C=3 (Quite sure: >80%)
Feedback
The submandibular salivary gland wraps itself around the free posterior border of the mylohyoid
muscle.The part of the gland that lies superficial to the mylohyoid is termed the
superficial 'lobe' while the part that lies deep to the mylohyoid is called the deep 'lobe'. It must be
noted that the so-called superficial and deep lobes are always physically continuous with each other
around the free posterior border of mylohyoid.
The hyoglossus, genioglossus, geniohyoid and stylohyoid muscles lie in a plane altogether deep to
the submandibular gland.
The correct answer is:
Mylohyoid
You did not select a certainty. Assuming: C=1 (Unsure: <67%).
Question 27
Correct
CBM mark 1.00
Weight 1.00
Flag question
Question text
Which one of the following arteries is a branch of the internal carotid artery?
Select one:
Infraorbital artery
Middle meningeal artery
Occipital artery
Ophthalmic artery
Superficial temporal artery
Certainty: C=1 (Unsure: <67%) C=2 (Mid: >67%) C=3 (Quite sure: >80%)
Feedback
The superficial temporal and occipital arteries are direct branches of the external carotid artery while
the middle meningeal and infraorbital arteries are branches of the maxillary artery, in turn a branch
of the external carotid artery.
The Ophthalmic artery is a direct branch of the internal carotid artery.
The correct answer is:
Ophthalmic artery
You did not select a certainty. Assuming: C=1 (Unsure: <67%).
Question 28
Correct
CBM mark 1.00
Weight 1.00
Flag question
Question text
A 35-year-old man attends the Emergency Department following a football tackle with a Weber C
ankle fracture (high fibula ankle fracture). He requires surgery and the fracture will be fixed with a lag
screw and neutralisation plate. Using the direct lateral approach to the fibula which of the following
nerves can be potentially injured passing across the fibula from posterior to anterior during your
approach?
Select one:
Deep peroneal nerve
Saphenous nerve
Superficial peroneal nerve
Sural nerve
Tibial nerve
Certainty: C=1 (Unsure: <67%) C=2 (Mid: >67%) C=3 (Quite sure: >80%)
Feedback
The superficial peroneal nerve runs from posterior to anterior across the fibula approximately 10cm
from the tip of the fibula as it passes distally to supply the skin sensation on the dorsum of the foot.
This is the nerve that is most at risk during the approach for this surgery.
The correct answer is:
Superficial peroneal nerve
You did not select a certainty. Assuming: C=1 (Unsure: <67%).
Question 29
Correct
CBM mark 1.00
Weight 1.00
Flag question
Question text
A 20-year-old woman underwent a right thoracoscopic sympathectomy for palmar hyperhidrosis. At
her follow-up appointment she reports satisfaction with her surgery but volunteers that the right side
of her face now feels warm and rather dry. She has also noted some drooping of her right upper
eyelid. Which one of the following physical signs is most likely to be present as well?
Select one:
Constricted right pupil
Divergent strabismus
Facial hyperaesthesia
Facial weakness
Normal cilio-spinal reflex
Certainty: C=1 (Unsure: <67%) C=2 (Mid: >67%) C=3 (Quite sure: >80%)
Feedback
Inadvertent damage to the cervical sympathetic chain leads to Horner’s syndrome which is
characterised by anhidrosis of the face, ptosis, loss of the cilio-spinal reflex and constriction of the
pupil. The pupil becomes constricted due to the unopposed action of the sphincter pupillae muscle,
innervated by parasympathetic fibres. In contrast the dilator pupillae muscle is supplied by the
cervical sympathetic chain. A functioning dilator pupillae muscle is required for the cilio-spinal reflex.
Ptosis is due to weakness of the smooth muscle component of levator palpebrae superioris which is
also supplied by the cervical sympathetic chain.
Horner syndrome is not associated with a squint or facial weakness as the external ocular and facial
muscles do not have motor supply from the sympathetic nervous system. As the cutaneous nerves
to the face via branches of the trigeminal nerve are not affected sensation is normal.
The correct answer is:
Constricted right pupil
You did not select a certainty. Assuming: C=1 (Unsure: <67%).
Question 30
Correct
CBM mark 1.00
Weight 1.00
Flag question
Question text
A 58-year-old woman presents with a sudden onset of severe headache, associated with nausea
and vomiting. On examination her GCS is 13/15. A CT scan is performed (see below). Analgesia
and fluids are prescribed. Urgent transfer to the neurosurgical unit is arranged. The neurosurgical
registrar advises that nimodipine is commenced. What is the mechanism of action of nimodipine?
Select one:
ACE inhibitor
Alpha blocker
Aquaporin
Beta Blocker
Calcium channel blocker
Certainty: C=1 (Unsure: <67%) C=2 (Mid: >67%) C=3 (Quite sure: >80%)
Feedback
Scan shows SAH. This is communicating rather than obstructive: all ventricles are seen on the scan
(lateral, 3rd and 4th) and are dilated. This excludes A, B and D. NPH is a form of communicating
hydrocephalus but the presence of the SAH and the raised pressure appearance on the scan (no
atrophy) excludes this diagnosis.
The correct answer is:
Calcium channel blocker
You did not select a certainty. Assuming: C=1 (Unsure: <67%).
You are assessing a 4-year-old boy in the urology clinic. The boy has recently moved to the UK and
has been referred by the General Practitioner. The parents are concerned that the child goes to the
toilet many times per day, doesn't like to pass urine standing up instead preferring to sit down, often
leaks urine on voiding and has a large tight gathering of foreskin on the top of the penis, but not all
the way round. Which one of the following is the most likely treatment?
Select one:
A mid-stream urine specimen and appropriate treatment for chronic UTI
A surgical reconstruction
Isolated circumcision
Reassurance
Referral to a behavioural psychologist
Certainty: C=1 (Unsure: <67%) C=2 (Mid: >67%) C=3 (Quite sure: >80%)
Feedback
Hypospadias affects 1:300 boys and is where the opening of the urethra (meatus) is not at the tip of
the penis but either further down or even at the base. The foreskin is gathered at the tip and often on
one side only rather than circumferentially.
Treatment is surgical, usually at 12 months of age, with reconstruction of the urethra together with
circumcision.
The correct answer is:
A surgical reconstruction
You did not select a certainty. Assuming: C=1 (Unsure: <67%).
Question 32
Incorrect
CBM mark 0.00
Weight 1.00
Flag question
Question text
You are called to the Emergency Department to review a 2-month-old boy with a 6 hour history of a
right groin swelling. The referring doctor says the patient is otherwise well and the abdomen is soft
and non tender. The doctor states the mass is in the right inguino-scrotal region, it does not
transilluminate but is not reducible. Which one of the following is your preferred management
option?
Select one:
Arrange an urgent ultrasound scan to establish the diagnosis
Give analgesia and attempt to reduce the mass
Keep the patient nil by mouth, place a nasogastric tube and start broad spectrum antibiotics
Schedule the patient for an elective operation in the next few weeks
Take the patient to theatre urgently for operative repair
Certainty: C=1 (Unsure: <67%) C=2 (Mid: >67%) C=3 (Quite sure: >80%)
Feedback
The history points to an incarcerated right inguinal hernia. There are no features in the vignette to
point to bowel obstruction. In paediatric practice, we would try to reduce the hernia with analgesia
(specifically morphine) and assuming this is successful, arrange a semi urgent herniotomy in 48
hours time once the oedema of the tissues in the operative site has settled.
A - this is a clinical diagnosis although imaging may occasionally be helpful in atypical cases
B - correct
C - NBM and NG tube would be indicated in bowel obstruction but attempted hernia reduction takes
precedence as it has a high rate of success
D - an incarcerated hernia cannot be left untreated as the risk is of bowel ischaemia and perforation.
A reducible hernia may be given an elective surgical date.
E - immediate surgery is avoided, as long as the hernia can be reduced. This may require consultant
input to achieve successful reduction
The correct answer is:
Give analgesia and attempt to reduce the mass
You did not select a certainty. Assuming: C=1 (Unsure: <67%).
Question 33
Correct
CBM mark 1.00
Weight 1.00
Flag question
Question text
A 25-year-old motorcyclist involved in an RTC has sustained blunt trauma to the left side of his
chest. He is tachypnoeic (RR 20/min) and has an oxygen saturation of 93% on air. He is
haemodynamically stable but his chest X-ray shows a large haemothorax. His heart size is normal
and the cardiac shadow is not displaced. Oxygen has been provided and intravenous access
obtained. In view of the amount of fluid seen in the left pleural cavity an intercostal drain is required.
Where is the most appropriate site for insertion of the drain?
Select one:
2nd interspace in the mid-clavicular line
2nd intercostal space mid-axillary line
5th interspace between the anterior axillary and mid-axillary lines
5th interspace in the mid-clavicular line
5th interspace posterior to the posterior axillary line
Certainty: C=1 (Unsure: <67%) C=2 (Mid: >67%) C=3 (Quite sure: >80%)
Feedback
The 5th interspace between the anterior axillary and mid-axillary lines is in the “safe area” and is the
preferred site for drain insertion.
The 5th interspace in the mid-clavicular line normally corresponds to the apex of the left ventricle
and therefore drain insertion here risks damage to the heart. Insertion of a drain posterior to the
posterior axillary line requires dissection through the latissimus dorsi muscle and a posteriorly sited
drain leads to discomfort for the patient when lying down. Insertion of a drain through the 8th
interspace risks injury to the diaphragm or even intraabdominal organs. In full expiration the dome
of the diaphragm reaches the 4th interspace on the right and the 5th interspace on the left side.
The 2nd interspace in the mid-clavicular line is used for urgent needle decompression of a tension
pneumothorax. Because of its position a drain at his site would not effectively drain a haemothorax.
The correct answer is:
5th interspace between the anterior axillary and mid-axillary lines
You did not select a certainty. Assuming: C=1 (Unsure: <67%).
Question 34
Correct
CBM mark 1.00
Weight 1.00
Flag question
Question text
A 22 year old male injures his knee while playing football and attends the emergency department.
He describes his injury occurring as he pivoted to suddenly change the direction he was running in, it
was a non contact injury. He reports immediate swelling in the knee and that he was unable to play
on. Which one of the following structures is he most likely to have injured?
Select one:
Anterior cruciate ligament
Lateral Meniscus
Medial collateral ligament
Medial meniscus
Posterior cruciate ligament
Certainty: C=1 (Unsure: <67%) C=2 (Mid: >67%) C=3 (Quite sure: >80%)
Feedback
From this history the most likely injury is an ACL rupture. The mechanism for an ACL rupture is
sudden change of direction or pivoting, and is usually a non contact injury common in football and
netball players. It produces a large haemarthrosis and therefore is associated with immediate knee
swelling and patients not being able to continue the sport or activity they were doing.
The correct answer is:
Anterior cruciate ligament
You did not select a certainty. Assuming: C=1 (Unsure: <67%).
Question 35
Correct
CBM mark 1.00
Weight 1.00
Flag question
Question text
A 19-year-old is admitted with a penetrating knife injury. His injuries are consistent with a severed
left half of his thoracic spinal cord at the T8 level. Which of the following signs is the most likely
consequence of transection of the left dorsal column?
Select one:
Loss of bladder control
Loss of light touch sensation on the left with an upper level 5 cm above the umbilicus
Loss of pinprick sensation below the umbilicus on the right
Loss of temperature sensation below the umbilicus on the right
Loss of vibration sense on the right
Certainty: C=1 (Unsure: <67%) C=2 (Mid: >67%) C=3 (Quite sure: >80%)
Feedback
T8 level approx 5 cm above umbilicus. Dorsal column loss causes ipsilateral loss below level of
injury - light touch, conscious proprioception, 2 point discrimination, vibration. Pain (pinprick) and
temperature loss are contralateral and start a couple of levels below the injury. They are
spinothalamic modalities NOT dorsal column. Bladder control may be lost but it is not a dorsal
column function.
The correct answer is:
Loss of light touch sensation on the left with an upper level 5 cm above the umbilicus
You did not select a certainty. Assuming: C=1 (Unsure: <67%).
Question 36
Correct
CBM mark 1.00
Weight 1.00
Flag question
Question text
A 41-year-old man presents with a history of right lower limb sciatica. Pain and numbness are
reported in the sole and lateral border of the foot. The ankle reflex is absent. At which one of the
following levels is the disc prolapse most likely to have occurred?
Select one:
L2/3
L3/4
L4/5
L5/S1
S1/S2
Certainty: C=1 (Unsure: <67%) C=2 (Mid: >67%) C=3 (Quite sure: >80%)
Feedback
Compression of the S1 nerve root accounts for the clinical features. The relation of the root to the
disc is such that an L5/S1 disc prolapse most commonly compresses the S1 root in isolation as in
the scenario.
The correct answer is:
L5/S1
You did not select a certainty. Assuming: C=1 (Unsure: <67%).
Question 37
Incorrect
CBM mark 0.00
Weight 1.00
Flag question
Question text
You are discussing post-operative pain relief during WHO checklist, for a patient about to undergo
an open nephrectomy through a loin incision above the 12th rib. Which one of the following methods
of post-operative / regional analgesia is most appropriate?
Select one:
Continual inter-costal infusion
Epidural anaesthesia
Infra-clavicular block
Patient Controlled Analgaesia (PCA)
Rectus sheath catheter
Certainty: C=1 (Unsure: <67%) C=2 (Mid: >67%) C=3 (Quite sure: >80%)
Feedback
PCA, epidural and inter-costal infusion are effective post-nephrectomy.
Recovery is faster with a local continuous infusion and allows earlier mobilisation and discharge.
The rectus sheets should not be entered during a loin approach to the kidney and so a rectus sheath
catheter is unhelpful.
The correct answer is:
Continual inter-costal infusion
You did not select a certainty. Assuming: C=1 (Unsure: <67%).
Question 38
Incorrect
CBM mark 0.00
Weight 1.00
Flag question
Question text
A 48 hour term, male infant presents with a 12 hour history of green vomiting. The parents state that
there have been some wet nappies but that the child has not opened his bowels since birth. The
abdomen is distended, full but not tense and not tender. The anus is normally sited. Which one of
the following is the most useful diagnostic test?
Select one:
Blood cultures
Lower gastrointestinal contrast study
Suction rectal biopsy
Ultrasound scan
Upper gastrointestinal contrast study
Certainty: C=1 (Unsure: <67%) C=2 (Mid: >67%) C=3 (Quite sure: >80%)
Feedback
The clinical history at this age and with the triad of bile vomiting, abdominal distension and failure to
pass meconium points to a likely diagnosis of Hirschsprung's disease. The diagnostic test is
therefore a suction rectal biopsy to look for an absence of ganglion cells. The normally sited anus
rules out an anorectal anomaly.
Suction rectal biopsy - correct
Upper gastrointestinal contrast study - the presence of bile vomiting may lead the candidate to
consider malrotation and volvulus but typically children with malrotation have passed meconium. If
this is felt to be ambiguous then an alternative answer option would be plain abdominal x-ray - this
would lead you to upper or lower GI obstruction but not to the underlying cause without further
investigation
Blood cultures - sepsis is a possible cause but less likely
Lower gastrointestinal contrast study - lower GI contrast can be used to investigate lower GI
obstruction but will not prove a diagnosis of Hirschsprung's disease
Ultrasound scan - USS is unlikely to be contributory here
The correct answer is:
Blood cultures
You did not select a certainty. Assuming: C=1 (Unsure: <67%).
Question 39
Correct
CBM mark 1.00
Weight 1.00
Flag question
Question text
You are assessing the neurovascular status in the lower limb of a motorcyclist with an open tibial
shaft fracture. There is significant soft tissue loss posteriorly and you are concerned about injury to
the posterior tibial artery. You are hopeful that the dorsalis pedis artery is spared as it lies between
which one of the following tendons?
Select one:
Between the extensor hallucis longus and extensor digitorum longus tendons
Between the flexor digitorum longus and the flexor halluces longus tendons
Between the flexor hallucis longus and flexor digitorum longus tendons
Between the tibialis anterior and extensor hallucis longus tendons
Between the tibialis posterior and flexor digitorum longus tendons
Certainty: C=1 (Unsure: <67%) C=2 (Mid: >67%) C=3 (Quite sure: >80%)
Feedback
Dorsalis pedis is the continuation of the anterior tibial artery after passing under the extensor
retinaculum. It is located on the dorsum of the foot, passing between the tendons of the extensor
hallucis longus and extensor digitorum longus.
The correct answer is:
Between the extensor hallucis longus and extensor digitorum longus tendons
You did not select a certainty. Assuming: C=1 (Unsure: <67%).
Question 40
Correct
CBM mark 1.00
Weight 1.00
Flag question
Question text
You are asked to assess a patient involved in a road traffic collision who sustained a multi-
fragmentary fracture of the distal femur. After considering the displacement forces in this type of
injury, which of the following anatomical structure are you most concerned about?
Select one:
Deep peroneal nerve
Popliteal artery
Quadriceps tendon
Superficial peroneal nerve
Tibial nerve
Certainty: C=1 (Unsure: <67%) C=2 (Mid: >67%) C=3 (Quite sure: >80%)
Feedback
Gastrocnemius originates from the supracondylar ridges of the distal femur and inserts via Achilles
tendon on the calcaneal tuberosity. In a distal femoral fracture, the heads of this muscle will displace
the distal fragment posteriorly, putting at risk the structures of the popliteal fossa. The deepest
structure of the popliteal fossa which lies in close proximity to the bone is popliteal artery.
The correct answer is:
Popliteal artery
You did not select a certainty. Assuming: C=1 (Unsure: <67%).
You are assessing a patient with an inguinal hernia. The patient has neurological symptoms
consistent with meralgia paresthetica as a result of a nerve intrapment into the inguinal ligament.
Which of the following is the trapped nerve?
Select one:
Intermediate cutaneous nerve of the thigh
Lateral cutaneous nerve of the thigh
Medial cutaneous nerve of the thigh
Posterior cutaneous nerve of the thigh
Saphenous nerve
Certainty: C=1 (Unsure: <67%) C=2 (Mid: >67%) C=3 (Quite sure: >80%)
Feedback
The lateral cutaneous nerve of the thigh (L2-L3) arises directly from the from the lumbar plexus and
usually enters the thigh deep to the inguinal ligament. Occasionally the nerve pierces the inguinal
ligament, is compressed here and gives pain and parasthesia over the superior aspect of the outer
thigh (meralgia paresthetica).
The correct answer is:
Lateral cutaneous nerve of the thigh
You did not select a certainty. Assuming: C=1 (Unsure: <67%).
Question 42
Correct
CBM mark 1.00
Weight 1.00
Flag question
Question text
You are assisting in placing a patient who has suffered a mid-shaft femoral fracture and multiple rib
fractures into femoral traction, as he is currently unfit for surgery. You position the traction to balance
the displacement of the proximal fragment because of which one of the following?
Select one:
Adduction by gracilis and flexion by sartorius
Flexion and adduction by the adductor magnus
Flexion by the iliopsoas muscle and abduction by the gluteus medius and minimus
Flexion by the iliopsoas muscle and adduction by the adductors
Flexion by the quadriceps and adduction by the semi-membranosus
Certainty: C=1 (Unsure: <67%) C=2 (Mid: >67%) C=3 (Quite sure: >80%)
Feedback
In a proximal femoral fracture the proximal fragment is flexed by the iliopsoas muscle (inserting on
the lesser trochanter) and abducted by the hip abductors: gluteus medius and minimus (inserting on
the greater trochanter). The distal fragment is adducted by the adductor insertion on the femoral
shaft.
The correct answer is:
Flexion by the iliopsoas muscle and abduction by the gluteus medius and minimus
You did not select a certainty. Assuming: C=1 (Unsure: <67%).
Question 43
Incorrect
CBM mark 0.00
Weight 1.00
Flag question
Question text
A rock climber slips for 3 meters on a fixed rope and hits the rock-face. Which of the following bones
is most likely to have suffered an open fracture?
Select one:
Femur
Humerus
Radius
Tibia
Ulna
Certainty: C=1 (Unsure: <67%) C=2 (Mid: >67%) C=3 (Quite sure: >80%)
Feedback
The tibial shaft is comparatively long, subcutaneous and unprotected anteromedially throughout its
entire course. It is also slender in its lower 1/3, making it very vulnerable to fractures in general. Due
to lack of a well-developed soft tissue envelope on the anteromedial aspect it is particularly prone to
open injuries.
The correct answer is:
Tibia
You did not select a certainty. Assuming: C=1 (Unsure: <67%).
Question 44
Correct
CBM mark 1.00
Weight 1.00
Flag question
Question text
Whilst on cardiothoracic placement, you are asked to harvest the long saphenous vein to be used for
a coronary artery bypass. How do you locate this structure at the level of the ankle?
Select one:
Behind the lateral malleolus
Behind the medial malleolus
Between the tendons of flexor digitorum longus and flexor hallucis longus
In front of the lateral mallolus
In front of the medial malleolus
Certainty: C=1 (Unsure: <67%) C=2 (Mid: >67%) C=3 (Quite sure: >80%)
Feedback
Correct 'in front of the medial malleolus peroneus longus': Long saphenous vein receives blood from
the medial aspect of the foot, passes approximately 2 cm in front of the medial malleolus with
saphenous nerve anterior to it and enters the femoral vein in the groin.
The correct answer is:
In front of the medial malleolus
You did not select a certainty. Assuming: C=1 (Unsure: <67%).
Question 45
Correct
CBM mark 1.00
Weight 1.00
Flag question
Question text
A 45-year-old man presents with acute lumbar back pain after picking up a heavy box. On
examination of his neurology, he has reduced power in dorsiflexion of his left hallux and reduced
sensation in the first web space of the left foot. Which of the following is the likely pathology causing
these signs and symptoms?
Select one:
Left sided far lateral disc prolapse at the L3/L4 level
Left sided far lateral disc prolaspe at the L4/L5 level
Left sided paracentral disc prolapse at the L3/L4 level
Left sided paracentral disc prolapse at the L4/L5 level
Left sided paracentral disc prolapse at the L5/S1 level
Certainty: C=1 (Unsure: <67%) C=2 (Mid: >67%) C=3 (Quite sure: >80%)
Feedback
The clinical examination shows reduced power and sensation in the L5 nerve root distribution. In the
lumbar spine at each disc level there will be a descending and exiting nerve root. The exiting nerve
root will be from the level above and the descending nerve root from the level below. For example at
the L4/L5 disc level there will be an exiting L4 nerve root and a descending L5 nerve root.
Descending nerve roots are affected by paracentral discs and exiting nerve roots by far lateral discs.
Therefore with symptoms suggesting L5 nerve root compression the options include a paracentral
disc at the L4/L5 level affecting the L5 descending nerve root or a far lateral disc at the level of
L5/S1 affecting the exiting L5 nerve root.
The correct answer is:
Left sided paracentral disc prolapse at the L4/L5 level
You did not select a certainty. Assuming: C=1 (Unsure: <67%).
Question 46
Correct
CBM mark 1.00
Weight 1.00
Flag question
Question text
A 7-year-old boy presents with a two month history of intermittent peri-umbilical pain and intermittent
presence of blood in his faeces. A tentative diagnosis of Meckel diverticulum is made. Which one of
the following is true about Meckel diverticulum?
Select one:
It arises on the mesenteric border of the ileum
It is a remnant of the vitelline duct
It is a remnant of the vitelline veins
It is located 60 cm distal to the ileocaecal valve
It is present in 20% of the population
Certainty: C=1 (Unsure: <67%) C=2 (Mid: >67%) C=3 (Quite sure: >80%)
Feedback
Meckel diverticulum is one of the most common congenital digestive system disorders. The
embryological origin is thought to be tissue from either the stomach or pancreas.
The correct answer is:
It is a remnant of the vitelline duct
You did not select a certainty. Assuming: C=1 (Unsure: <67%).
Question 47
Correct
CBM mark 1.00
Weight 1.00
Flag question
Question text
A patient with recent history of headaches and unsteadiness is found to have a pre-pontine
meningioma arising from the clivus. The patient also reports diplopia. Compression of which of the
following nerve(s) is most likely to account for the diplopia?
Select one:
Abducens
Oculomotor
Ophthalmic division of trigeminal
Sympathetic
Trochlear
Certainty: C=1 (Unsure: <67%) C=2 (Mid: >67%) C=3 (Quite sure: >80%)
Feedback
A clival tumour is most likely to compress the VI nerve as it ascends from the pontomedullary
junction immediately posterior to the clivus. The other nerves are not closely related to the clivus and
the ophthalmic division of V and sympathetic do not control eye movements and therefore do not
cause diplopia.
The correct answer is:
Abducens
You did not select a certainty. Assuming: C=1 (Unsure: <67%).
Question 48
Correct
CBM mark 1.00
Weight 1.00
Flag question
Question text
A 40-year-old amateur soprano singer recently underwent a subtotal thyroidectomy for Graves'
disease. When starting back in her choir two weeks after surgery, her fellow members commented
that the pitch of her voice had changed. In addition, she noted that she was unable to reach the
higher notes that she used to be able to sing quite easily before her operation. Which of the following
nerves is most likely to have been injured during her surgery?
Select one:
External laryngeal nerve
Inferior root of the ansa cervicalis
Internal laryngeal nerve
Recurrent laryngeal nerve
Superior root of the ansa cervicalis
Certainty: C=1 (Unsure: <67%) C=2 (Mid: >67%) C=3 (Quite sure: >80%)
Feedback
The external laryngeal nerve which is a branch of the superior laryngeal nerve supplies motor fibres
to the crico-thyroid muscle. This muscle tenses the vocal cords and is responsible for the quality and
pitch of the voice.
In contrast the internal laryngeal nerve, the other branch of the superior laryngeal nerve, is a purely
sensory nerve supplying the mucosa of the larynx and pharynx.
The recurrent laryngeal nerve supplies the intrinsic muscles of the larynx and injury leads to a either
a hoarse voice or even aphonia, if bilateral injury occurs.
The ansa cervicalis supplies the following infrahyoid muscles which are depressors of the larynx:
sterno-hyoid, omo-hyoid, sterno-thyroid. These muscles are not directly involved in voice production.
The correct answer is:
External laryngeal nerve
You did not select a certainty. Assuming: C=1 (Unsure: <67%).
Question 49
Correct
CBM mark 1.00
Weight 1.00
Flag question
Question text
A 55-year-old woman has undergone right mastectomy and level 3 axillary node clearance for a 5
cm node positive, grade 3 invasive ductal carcinoma. The pectoralis minor muscle was divided at its
insertion into the coracoid process of the scapula to facilitate removal of enlarged lymph nodes at
level 3. Post operation, she complains of numbness and paraesthesia of the medial aspect of her
upper arm. Which of the following nerves has been injured during the operation?
Select one:
Axillary nerve
Intercostal-brachial nerve
Lateral pectoral nerve
Long thoracic nerve
Musculo-cutaneous nerve
Certainty: C=1 (Unsure: <67%) C=2 (Mid: >67%) C=3 (Quite sure: >80%)
Feedback
The intercosto-brachial nerve (T2) is the lateral cutaneous branch of the second intercostal nerve. It
supplies the skin of the axilla and the skin of the medial aspect of the upper arm close to the
axilla. It is at risk of injury during extensive axillary surgery
The axillary nerve (C5) supplies the skin of the lateral aspect of the upper arm mainly via the upper
lateral cutaneous nerve of the arm.
The lateral pectoral nerve (C6,7) is a motor nerve only, supplying the pectoralis major muscle.
The long thoracic nerve (C5,6,7) is a motor nerve only supplying the serratus anterior muscle.
The musculo-cutaneous nerve (C5,6,7) is a motor nerve to biceps, brachialis and coraco-brachialis
and forms the lateral cutaneous nerve of the forearm.
The correct answer is:
Intercostal-brachial nerve
You did not select a certainty. Assuming: C=1 (Unsure: <67%).
Question 50
Correct
CBM mark 1.00
Weight 1.00
Flag question
Question text
A 20-year-old man has undergone recent excision biopsy of an enlarged lymph node situated in the
posterior triangle of the right side of his neck. The procedure was performed under general
anaesthetic. Histology has shown nodular sclerosing Hodgkin's lymphoma. When seen seven days
later for the results of the biopsy the patient complains that he has weakness in the region of his
right shoulder and he is unable to shrug his shoulder on that side. Injury to which one of the following
nerves is likely to account for the patient's symptoms and signs?
Select one:
Accessory nerve
Intermediate supraclavicular nerve
Lateral (posterior) supraclavicular nerve
Medial supraclavicular nerve
Transverse cervical nerve
Certainty: C=1 (Unsure: <67%) C=2 (Mid: >67%) C=3 (Quite sure: >80%)
Feedback
The accessory nerve supplies motor fibres to the trapezius muscle, contraction of which elevates the
scapula and the lateral end of the clavicle.
The other nerves are all sensory nerves supplying the skin of the neck.
The correct answer is:
Accessory nerve
You did not select a certainty. Assuming: C=1 (Unsure: <67%).
A 9-month-old woman, born four weeks prematurely, is presented with a history of poor growth,
frequent lethargy and a persistent tachycardia. Auscultation of the chest reveals a systolic murmur,
and there is a 'waterhammer' pulse. Blood pressure in the upper and lower limbs are equal and
normal. Breathing rate is normal for age. What is the likely diagnosis?
Select one:
Atrial septal defect
Coarctation of the aorta
Patent ductus arteriosus
Transposition of the great arteries (TGA)
Truncus arteriosus
Certainty: C=1 (Unsure: <67%) C=2 (Mid: >67%) C=3 (Quite sure: >80%)
Feedback
Patent ductus arteriosus is associated with systolic murmur and waterhammer pulse.
Coarctation of the aorta often produces differential blood pressure between upper and lower limbs.
ASD would give a diastolic murmur.
TGA presents differently due to reversal of the great arteries leaving the heart.
Truncus arteriosus presents much earlier with surgery often required by 3 months of age and
breathlessness is often a feature.
The correct answer is:
Patent ductus arteriosus
You did not select a certainty. Assuming: C=1 (Unsure: <67%).
Question 52
Correct
CBM mark 1.00
Weight 1.00
Flag question
Question text
An 18-year-old man arrives as a trauma call, he has crashed his motorbike at high speed and hit a
bollard head on. He is tachycardic, hypotensive and has bilateral externally rotated legs. Which one
of the following pelvic fractures is most likely with the patient's history and presentation?
Select one:
Acetabular fracture
Anterior posterior compression fracture (open book)
Fractured neck of femur
Lateral compression fracture
Vertical sheer fracture
Certainty: C=1 (Unsure: <67%) C=2 (Mid: >67%) C=3 (Quite sure: >80%)
Feedback
The mechanism of injury here is anterior- posterior compression from the petrol tank of the motor
bike that would get forced up into the pelvis with a head on collision. These fractures are associated
with haemodynamic instability. As the injury involves the pelvis opening up at the front clinically the
legs can assume an externally rotated position. A lateral compression and acetabular fracture is
associated with impact from one side. The mechanism for a vertical sheer fracture is normally a fall
from height, although it is often associated with haemodynamic instability.
The correct answer is:
Anterior posterior compression fracture (open book)
You did not select a certainty. Assuming: C=1 (Unsure: <67%).
Question 53
Incorrect
CBM mark 0.00
Weight 1.00
Flag question
Question text
You are called to the post-natal ward to see a 5-day-old girl who was born 14 days prematurely. The
paediatric team had noticed an erythematous inflamed area between the genitalia and the anus, with
evidence of localised inflammation. The fluid chart has documented very loose stools, green/brown
in colour. The baby is otherwise feeding normally. Which one of the following is the most likely
diagnosis?
Select one:
Fistula
Hirschsprung's disease
Hypospadias
Perineal groove
Prolapsed anus
Certainty: C=1 (Unsure: <67%) C=2 (Mid: >67%) C=3 (Quite sure: >80%)
Feedback
Perineal groove is a rare anomaly that occurs primarily in female infants. It is thought to be either a
failure of midline fusion or a urorectal septum developmental defect during cloacal embryological
stages at 5th to 8th week of gestation.
In many cases the area self-epithelialise, though observation is needed to prevent infection.
Hirschsprung's presents with constipation or failure to pass faeces.
A fistula should form part of your differential, but would be an obvious communication leaking bowel
content.
Hypospadias occurs in male infants.
The correct answer is:
Perineal groove
You did not select a certainty. Assuming: C=1 (Unsure: <67%).
Question 54
Correct
CBM mark 1.00
Weight 1.00
Flag question
Question text
You assess an elderly patient who has been diagnosed with a pharyngeal pouch. This is an out
pouching of the oesophagus through an area called Killian dehiscence between which muscles of
the pharynx?
Select one:
Inferior constrictor and cricopharyngeus
Salpingopharyngeus and inferior constrictor
Salpingopharyngeus and superior constrictor
Superior and inferior constrictors
Superior constrictor and cricopharyngeus
Certainty: C=1 (Unsure: <67%) C=2 (Mid: >67%) C=3 (Quite sure: >80%)
Feedback
Killian dehiscence is between the inferior constrictor and cricopharynxgeus. Aetiology of pharyngeal
pouch is thought to be related to dysfunction of the cricopharyngeus.
The correct answer is:
Inferior constrictor and cricopharyngeus
You did not select a certainty. Assuming: C=1 (Unsure: <67%).
Question 55
Incorrect
CBM mark 0.00
Weight 1.00
Flag question
Question text
A 60-year-old woman undergoes surgery to clip a posterior inferior cerebellar artery aneurysm.
Cranial nerve injury may occur during the surgical exposure. Which one of the following deficits is
most commonly associated with this operation post-operatively?
Select one:
Diplopia
Facial weakness and numbness
Hoarse voice/weak cough
Tongue atrophy
Visual field defect
Certainty: C=1 (Unsure: <67%) C=2 (Mid: >67%) C=3 (Quite sure: >80%)
Feedback
The cranial nerves intimately related to the PICA vessel and therefore vulnerable to injury during this
procedure are IX and X. This causes hoarse voice/weak cough
The correct answer is:
Hoarse voice/weak cough
You did not select a certainty. Assuming: C=1 (Unsure: <67%).
Question 56
Correct
CBM mark 1.00
Weight 1.00
Flag question
Question text
A 50-year-old-woman presents to the upper GI clinic with a six month history of sporadic central
abdominal pain. She also suffers mild dyspepsia. She has been a lifetime heavy smoker and over
the last 18 months has had investigations for peripheral vascular disease. Her pain only arises after
eating, especially larger meals. Which one of the following is the most likely cause for her
symptoms?
Select one:
Barrets oesophagus
Biliary cholic
Irritable bowel disease
Oesophageal spasm
Superior mesenteric stenosis
Certainty: C=1 (Unsure: <67%) C=2 (Mid: >67%) C=3 (Quite sure: >80%)
Feedback
The history of post-prandial abdominal pain is indicative of vascular insufficiency to the gut,
especially given the history of smoking and recent peripheral vascular disease.
The correct answer is:
Superior mesenteric stenosis
You did not select a certainty. Assuming: C=1 (Unsure: <67%).
Question 57
Correct
CBM mark 1.00
Weight 1.00
Flag question
Question text
A 32-year-old patient is undergoing an emergency laparotomy following blunt trauma to the
abdomen. On opening the peritoneum there is torrential bleeding and your consultant performs
'Pringle's manoeuvre'. Which one of the following structures are clamped by this manoeuvre?
Select one:
Gastroduodenal artery and celiac trunk
Hepatic artery and portal vein
Inferior vena cava and portal vein
Splenic artery and portal vein
Superior vena cava and hepatic artery
Certainty: C=1 (Unsure: <67%) C=2 (Mid: >67%) C=3 (Quite sure: >80%)
Feedback
First described by J. Pringle, this 'manoeuvre' essentially prevents inflow of blood to the liver by
compressing the hepatic artery and portal vein. Performed either manually or with a clamp, it helps
in controlling hepatic haemorrhage.
The common bile duct is also compressed by this action.
The correct answer is:
Hepatic artery and portal vein
You did not select a certainty. Assuming: C=1 (Unsure: <67%).
Question 58
Incorrect
CBM mark 0.00
Weight 1.00
Flag question
Question text
A 25-years-old motorcyclist has been admitted to the Emergency Department following a road traffic
collision. His only significant injury is that of a probable fracture of the right humerus which is
confirmed by X-ray showing a displaced fracture of the mid shaft of the humerus. He complains of
weakness in the right arm and hand and appears to have a wrist drop. Which of the following
combinations of movements is most likely to be impaired?
Select one:
Extension of the elbow and wrist joints
Extension of the elbow, wrist and metacarpo-phalangeal joints
Extension of the metacarpo-phalangeal and interphalangeal joints
Extension of the wrist and metacarpo-phalangeal joints
Extension of the wrist, metacarpo-phalangeal and interphalangeal joints
Certainty: C=1 (Unsure: <67%) C=2 (Mid: >67%) C=3 (Quite sure: >80%)
Feedback
The patient has suffered injury to the radial nerve where it courses through the spiral groove of the
humerus. The nerve supply to the triceps muscle from the radial nerve is at a more proximal level
and so elbow extension is preserved. The extensor muscles of the wrist joint such as extensor carpi
radialis (longus and brevis) and extensor carpi ulnaris will be affected by the injury. Similarly, the
muscles extending the metacarpo-phalangeal joints joints such as extensor digitorum will be
weakened. However, extension of the interphalangeal joints can still occur due the action of the
lumbrical muscles which are supplied buy the ulnar and median nerves.
The correct answer is:
Extension of the wrist and metacarpo-phalangeal joints
You did not select a certainty. Assuming: C=1 (Unsure: <67%).
Question 59
Incorrect
CBM mark 0.00
Weight 1.00
Flag question
Question text
A 12-year-old girl presents with a 2 cm diameter anterior midline cyst of the neck. It moves on
swallowing and on protrusion of her tongue. At operation the cyst is very closely associated with a
bone derived from two of the embryological pharyngeal arches. From which one of the following
pharyngeal arches is this bone derived?
Select one:
1st and 2nd arches
2nd and 3rd arches
3rd and 4th arches
4th and 5th arches
5th and 6th arches
Certainty: C=1 (Unsure: <67%) C=2 (Mid: >67%) C=3 (Quite sure: >80%)
Feedback
The clinical features are typical of a thyroglossal cyst which is derived from remnants of the
thyroglossal duct. Embryologically this duct passes close to the anterior part of the body of the hyoid
bone, often also passing posterior to the bone before descending towards the thyroid gland. The
hyoid bone is derived from the 2nd and 3rd pharyngeal arches.
The 1st arch forms the maxilla and mandible. The 4th and 6th arches form the muscles and cartilage
of the larynx whilst the 5th arch usually disappears.
The correct answer is:
2nd and 3rd arches
You did not select a certainty. Assuming: C=1 (Unsure: <67%).
Question 60
Incorrect
CBM mark 0.00
Weight 1.00
Flag question
Question text
You see a 34-year-old woman in the Emergency Department following a road traffic collision. The
ambulance crew reported a Glasgow Coma Scale (GCS) of 9 with pupils that were equally reactive
to light. You record her GCS as 5 and note that she now has a fixed dilated pupil on the left. A CT
scan is performed. Which one of the following is the clinically most urgent finding on this scan?
Select one:
Acute subdural haematoma
Extradural haematoma
Skull base fracture
Temporal lobe contusion
Traumatic subarachnoid haemorrhage
Certainty: C=1 (Unsure: <67%) C=2 (Mid: >67%) C=3 (Quite sure: >80%)
Feedback
All 5 of the options are shown on this trauma scan. The obvious abnormality is the mass on the left
side (using standard radiology convention for side). This has the classic lens shape. Although not
particularly large it accounts for the dilated pupil on the left and warrants emergency surgery.
The correct answer is:
Extradural haematoma
You did not select a certainty. Assuming: C=1 (Unsure: <67%).
An 8- year-old man presents to clinic via the 2-week wait pathway with a complete right lower motor
neurone facial paralysis and a painless mass in his ipsilateral parotid gland. Which one of the
following is the most likely histological diagnosis?
Select one:
Acinic cell carcinoma
Adenoid cystic carcinoma
Carcinoma ex-pleomorphic adenoma
Lymphoma
Squamous cell carcinoma
Certainty: C=1 (Unsure: <67%) C=2 (Mid: >67%) C=3 (Quite sure: >80%)
Feedback
25% of salivary gland malignancy presents in the parotid gland (50% in the submandibular and 75%
in minor salivary glands). The presence of a parotid mass mass with a facial nerve paralysis is highly
suspicious for malignancy. The commonest malignant parotid tumour is adenoid cystic carcinoma
(approx. 25%), and poorly differentiated carcinoma (approx. 25%), then carcinoma ex-pleomorphic
(approx. 10%), then acinic cell carcinoma (3%).
The correct answer is:
Adenoid cystic carcinoma
You did not select a certainty. Assuming: C=1 (Unsure: <67%).
Question 62
Correct
CBM mark 1.00
Weight 1.00
Flag question
Question text
A 54-year-old man returns from a ski holiday. Following a fall onto the outstretched hand he
developed pain lasting about 10 minutes in his 3rd digit (middle finger). He immediately noticed that
his distal phalanx was flexed at the DIP joint. He was unable to voluntarily extend the DIP, although
he could passively straighten the finger without difficulty. An X-ray showed no evidence of a fracture.
What is the name of this injury?
Select one:
Boutonniere deformity
Hammer finger
Mallet finger
Swan neck deformity
Trigger finger
Certainty: C=1 (Unsure: <67%) C=2 (Mid: >67%) C=3 (Quite sure: >80%)
Feedback
This is a classical description of a mallet finger. The distal attachment of the extensor tendon is
avulsed from the terminal phalanx. Distractors all affect the fingers but are incorrect.
The correct answer is:
Mallet finger
You did not select a certainty. Assuming: C=1 (Unsure: <67%).
Question 63
Incorrect
CBM mark 0.00
Weight 1.00
Flag question
Question text
A 25-year-old woman falls from a horse and sustains a cervical spine injury. On arrival at the
Emergency Department, she is assessed and stabilised as per the ATLS principles. During the
secondary surgery examination she has 5/5 power in elbow flexion and 0/5 power below this level
bilaterally. Sensation is normal on the lateral aspect of the arm over the deltoid region down to the
elbow but is abnormal from the elbow down to the hand bilaterally. Perianal sensation is intact. How
would you describe the spinal cord injury?
Select one:
C4 Complete injury
C4 Incomplete injury
C5 Complete injury
C5 Incomplete injury
C6 Incomplete injury
Certainty: C=1 (Unsure: <67%) C=2 (Mid: >67%) C=3 (Quite sure: >80%)
Feedback
To determine the single neurological level you must work out the lowest segment where motor and
sensory function is normal bilaterally. This patient has normal elbow flexion (C5) and sensation over
the lateral aspect of the arm down to the elbow (C5) and so the sensory and motor level is C5.
Perianal sensation is preserved and so the injury is incomplete.
The correct answer is:
C5 Incomplete injury
You did not select a certainty. Assuming: C=1 (Unsure: <67%).
Question 64
Correct
CBM mark 1.00
Weight 1.00
Flag question
Question text
A 25-year-old motorcyclist is admitted following a high speed road traffic collision. He is paraplegic
with a sensory level at T5. He is noted to have priapism. Which one of the following is the most likely
explanation of the priapism?
Select one:
Excitation of the sympathetic trunk
Loss of inhibition of the parasympathetic nervi erigentes
Loss of inhibition of the sympathetic output
Pudendal nerve lesion
Sacral root lesion
Certainty: C=1 (Unsure: <67%) C=2 (Mid: >67%) C=3 (Quite sure: >80%)
Feedback
Priapism is due to loss of descending inhibition of sacral parasympathetic output. Other distractors
are reasonable considerations but all are incorrect.
The correct answer is:
Loss of inhibition of the parasympathetic nervi erigentes
You did not select a certainty. Assuming: C=1 (Unsure: <67%).
Question 65
Incorrect
CBM mark 0.00
Weight 1.00
Flag question
Question text
A 68-year-old woman presents with symptoms consistent with a prolapsed cervical disc. She reports
pain and numbness in the right lateral forearm, thenar eminence and thumb. The biceps and
supinator reflexes are absent. At which level is the prolapsed disc most likely to have occurred?
Select one:
C3/4
C4/5
C5/6
C6/7
C7/T1
Certainty: C=1 (Unsure: <67%) C=2 (Mid: >67%) C=3 (Quite sure: >80%)
Feedback
The description of symptoms is characteristic of C6 root compression. This is most commonly
compressed by a prolapsed C5/6 disc due to the relationship between nerve root and the disc.
The correct answer is:
C5/6
You did not select a certainty. Assuming: C=1 (Unsure: <67%).
Question 66
Correct
CBM mark 1.00
Weight 1.00
Flag question
Question text
A 65-year-old farmer presents to the Emergency Department after losing control of a circular saw
and sustaining a laceration posterior to the medial malleolus. You examine the wound under local
anaesthetic to identify if any structures have been damaged. When working from anterior to
posterior, which is the correct order of structures passing behind the medial malleolus?
Select one:
Extensor digitorum longus tendon, posterior tibial artery, tibial nerve, extensor hallucis longus tendon
and tibialis posterior tendon
Flexor digitorum longus tendon, posterior tibial artery, tibial nerve, flexor hallucis longus tendon and
tibialis posterior tendon
Flexor digitorum longus tendon, tibialis posterior tendon, posterior tibial artery, tibial nerve and flexor
hallucis longus tendon
Tibialis posterior tendon, flexor digitorum longus tendon, posterior tibial artery, tibial nerve and flexor
hallucis longus tendon
Tibialis posterior tendon, posterior tibial artery, tibial nerve, flexor digitorum longus tendon and flexor
hallucis longus tendon
Certainty: C=1 (Unsure: <67%) C=2 (Mid: >67%) C=3 (Quite sure: >80%)
Feedback
The correct order is option D, when considering the structures responsible for flexing/ extending the
toes, flexors run behind the medial malleolus and the extensors pass anterior to the ankle joint.
The correct answer is:
Tibialis posterior tendon, flexor digitorum longus tendon, posterior tibial artery, tibial nerve and flexor
hallucis longus tendon
You did not select a certainty. Assuming: C=1 (Unsure: <67%).
Question 67
Incorrect
CBM mark 0.00
Weight 1.00
Flag question
Question text
A 4-year-old boy sustains a closed extension-type supracondylar fracture of the humerus. Which one
of the following nerves is most likely to suffer a neuropraxia?
Select one:
Anterior interosseous nerve
Axillary nerve
Posterior interosseous nerve
Radial nerve
Ulna nerve
Certainty: C=1 (Unsure: <67%) C=2 (Mid: >67%) C=3 (Quite sure: >80%)
Feedback
The anterior interosseous nerve is most commonly injured in extension type injuries. Posterior
interosseous injury is usually associated with radial head fractures. Ulnar neuropraxia (not listed) is
associated with flexion-type injuries; ulna nerve direct injury is commonly associated with surgical
fine-wire insertion.
The correct answer is:
Anterior interosseous nerve
You did not select a certainty. Assuming: C=1 (Unsure: <67%).
Question 68
Correct
CBM mark 1.00
Weight 1.00
Flag question
Question text
A sciatic nerve block is performed just above the popliteal fossa for a patient who is about to
undergo ankle fracture fixation under general anaesthetic. Which one of the following areas is likely
to have normal sensation post-operatively?
Select one:
1st dorsal webspace
Anterolateral aspect of the foot
Dorsomedial aspect of the foot
Lesser toes
Sole of the foot
Certainty: C=1 (Unsure: <67%) C=2 (Mid: >67%) C=3 (Quite sure: >80%)
Feedback
The block will affect branches of the sciatic nerve, but will not affect the terminal branch of the
femoral nerve, namely the saphenous distribution providing sensation along the medial border of the
foot.
The correct answer is:
Dorsomedial aspect of the foot
You did not select a certainty. Assuming: C=1 (Unsure: <67%).
Question 69
Incorrect
CBM mark 0.00
Weight 1.00
Flag question
Question text
You have examined a 12-month-old child in paediatric surgical clinic with a diagnosis of syndactaly
of the fingers. You are explaining the diagnosis and history to the parents. Which of the following is
correct?
Select one:
Occurs in 1:1,000, caucasian male preponderance, sporadic inheritance pattern, usually unilateral,
commonly affects the 3rd webspace
Occurs in 1:3,000, asian female preponderance, autosomal recessive inheritance, usually unilateral,
commonly affects the 1st webspace
Occurs in 1:3,000, caucasian male preponderance, autosomal dominant inheritance, often bilateral,
commonly affects the 3rd webspace
Occurs in 1:3,000, caucasian male preponderance, autosomal dominant inheritance, usually unilateral,
commonly affects the 1st webspace
Occurs in 1:30,000, asian female preponderance, autosomal dominant inheritance, often bilateral,
commonly affects the 3rd webspace
Certainty: C=1 (Unsure: <67%) C=2 (Mid: >67%) C=3 (Quite sure: >80%)
Feedback
Occurs in 1:3,000
More common in caucasians rather than black or asian.
More common in males.
Autosomal dominant inheritance, often bilateral.
Commonly affects the 3rd webspace, rare in the first
The correct answer is:
Occurs in 1:3,000, caucasian male preponderance, autosomal dominant inheritance, often bilateral,
commonly affects the 3rd webspace
You did not select a certainty. Assuming: C=1 (Unsure: <67%).
Question 70
Correct
CBM mark 1.00
Weight 1.00
Flag question
Question text
A 26-year-old woman presents with a cough headache, tingling in the hands, and loss of pinprick
and temperature sensation in the hands. An MRI scan shows downward herniation of the cerebellar
tonsils and mid-cervical syringomyelia. Which of the following tracts is most likely to malfunction as a
result of this pathology?
Select one:
Corticospinal
Dorsal columns
Spinocerebellar
Spinothalamic
Vestibulospinal
Certainty: C=1 (Unsure: <67%) C=2 (Mid: >67%) C=3 (Quite sure: >80%)
Feedback
Herniation of tonsils compresses the craniocervical junction and cases syringomyelia. The
spinothalamic tracts cross in region of central canal - these are vulnerable to damage when a syrinx
expands. Typically causes tingling, burns on hands in a "suspended" or cape-like distribution.
The correct answer is:
Spinothalamic
You did not select a certainty. Assuming: C=1 (Unsure: <67%).
A patient suffers injury to the common peroneal nerve following a high fibular fracture. After six
months, recovery has been minimal and she complains of persistent foot-drop. Having failed
conservative measures, which one of the following tendon transfer procedures would be
appropriate?
Select one:
Extensor digitorum longus
Extensor hallucis longus
Peroneus brevis
Peroneus longus
Tibialis posterior
Certainty: C=1 (Unsure: <67%) C=2 (Mid: >67%) C=3 (Quite sure: >80%)
Feedback
The only tendon unaffected by a common peroneal nerve injury is the tibialis posterior tendon.
Despite being a plantar flexor invertor, it can be transferred into the anterior compartment to act as
an ankle dorsiflexor.
The correct answer is:
Tibialis posterior
You did not select a certainty. Assuming: C=1 (Unsure: <67%).
Question 72
Correct
CBM mark 1.00
Weight 1.00
Flag question
Question text
An 18-year-old female motorcyclist is involved in a high-speed road traffic accident. Primary survey
reveals a patient in respiratory distress with tracheal deviation towards the left. Breath sounds in the
right side of the chest are reduced.
What is the most appropriate next step in assessment and treatment of this patient?
Select one:
Left sided tube intercostal thoracostamy
Needle pericardiocentesis
Right sided needle decompression thoracostomy
Urgent chest radiograph
Urgent CT with possible interventional radiology
Certainty: C=1 (Unsure: <67%) C=2 (Mid: >67%) C=3 (Quite sure: >80%)
Feedback
This is a RIGHT sided tension pneumothorax; the breath sounds are reduced on the right and the
trachea is being pushed towards the left. The right side of the chest requires de-tensioning.
Decompression of the left side will worsen the clinical situation.
The correct answer is:
Right sided needle decompression thoracostomy
You did not select a certainty. Assuming: C=1 (Unsure: <67%).
Question 73
Correct
CBM mark 1.00
Weight 1.00
Flag question
Question text
A 75-year-old woman attends to have a left shoulder replacement. A delto-pectoral approach is the
approach the surgeon will be using and is a true inter-nervous plane. Which one of the following is
the inter-nervous plane?
Select one:
Axillary nerve and medial and lateral pectoral nerves
Axillary nerve and medial pectoral nerve
Lateral pectoral nerve and musculotanous nerve
Musculotanous nerve and radial nerve
Radial nerve and Axillary nerve
Certainty: C=1 (Unsure: <67%) C=2 (Mid: >67%) C=3 (Quite sure: >80%)
Feedback
The delto-pectoral approach is between the deltoid muscle and the pectoralis major muscle. The
nerve supplying the deltoid is the axillary nerve and the nerves supplying the pectoralis major
muscle are the medial an lateral pectoral nerves.
Pectoralis minor is supplied just by the medial pectoral nerve.
The correct answer is:
Axillary nerve and medial and lateral pectoral nerves
You did not select a certainty. Assuming: C=1 (Unsure: <67%).
Question 74
Incorrect
CBM mark 0.00
Weight 1.00
Flag question
Question text
You see a 34-year-old man in the Emergency Department following a road traffic collision. The
ambulance crew reported a Glasgow Coma Score (GCS) of 9 with pupils that were equally reactive
to light. You record his GCS as 5 and note that he now has a fixed dilated pupil on the right. Which
one of the following is the most likely explanation for the pupillary dilatation?
Select one:
Compression of abducens nerve
Direct compression of the midbrain
Disruption of ascending sympathetic fibres caused by a carotid artery
Ischaemia of the Edinger-Westphal nucleus
Loss of parasympathetic innervation of the pupil
Certainty: C=1 (Unsure: <67%) C=2 (Mid: >67%) C=3 (Quite sure: >80%)
Feedback
The pupil is innervated by autonomic nerve fibres. Parasympathetic fibres cause pupillary
constriction, travelling from the Edinger–Westphal nuclei in the midbrain via cranial nerve III to the
pupilloconstrictor muscle. Increased intracranial pressure may cause compromise of this pupillary
response and is one of the few signs detectable in the unconscious patient.
The correct answer is:
Loss of parasympathetic innervation of the pupil
You did not select a certainty. Assuming: C=1 (Unsure: <67%).
Question 75
Incorrect
CBM mark 0.00
Weight 1.00
Flag question
Question text
A 25-year-old motorcyclist presents to the Emergency Department at 23:00 hours with an isolated
high energy open fracture of the left tibia. He was wearing full protective clothing and was not in an
agricultural or aquatic environment. On examination he has a 7 cm laceration overlying the
subcutaneous border of the tibia which is visible and there is soft tissue loss. His pulses are intact
distally and radiographs show a spiral fracture of the tibial mid shaft. IV antibiotics have been given
and tetanus status checked. Which one of the following is the most appropriate management?
Select one:
Apply a saline soaked gauze and occlusive dressing to the wound and splint the limb, after manipulation
if indicated. Aim for theatre for a joint orthoplastics procedure at the next available opportunity within
12 hours.
Prepare the patient to be taken to theatre immediately for lower limb fasciotomies to decompress
compartments as this injury is high risk for compartment syndrome.
Prepare the patient to be taken to theatre immediately for washout and debridement of the wound and
application of an external fixator by the orthopaedic team.
Wash the wound in ED with sterile saline and dress it with a betadine soaked gauze. Aim for theatre for
a joint orthoplastics procedure at the next available opportunity within 12 hours.
Washout the wound in ED with sterile saline and dress with a betadine soaked gauze. Splint the limb,
after manipulation if indicated. Aim for theatre for a joint orthoplastics procedure within the next 24
hours.
Certainty: C=1 (Unsure: <67%) C=2 (Mid: >67%) C=3 (Quite sure: >80%)
Feedback
The British Orthopaedic Association and British Association of Plastic Reconstructive and Aesthetic
Surgeons joint guideline for open fractures states that intravenous antibiotics should be administered
ideally within 1 hour of injury. Wounds should only be handled to remove gross contamination,
dressed with saline soaked gauze and covered with an occlusive film. Washouts in the emergency
department are not indicated, nor is betadine soaked dressings.
Indications for immediate theatre intervention include highly contaminated wounds (agricultural,
aquatic, sewage), arterial injuries and signs of compartment syndrome. For solitary high energy
open fractures, the aim is to be in theatre within 12 hours of injury. Initial surgery should be carried
out by consultants in orthopaedics and plastic surgery - a combined orthoplastic approach.
The correct answer is:
Apply a saline soaked gauze and occlusive dressing to the wound and splint the limb, after
manipulation if indicated. Aim for theatre for a joint orthoplastics procedure at the next available
opportunity within 12 hours.
You did not select a certainty. Assuming: C=1 (Unsure: <67%).
Question 76
Correct
CBM mark 1.00
Weight 1.00
Flag question
Question text
You are asked to review a 50-year-old medical secretary with progressive, radial sided wrist pain.
On examination, the pain is worse on ulnar deviation yet range of movement of the wrist joint is
good. Radiographs were unremarkable. You suspect a De Quervain's tenosynovitis because of
inflammation in the dorsal compartment containing which of the following tendons?
Select one:
Extensor carpi radialis longus and brevis
Extensor carpi ulnaris
Extensor indicis and extensor digitorum communis
Extensor pollicis brevis and abductor pollicis longus
Extensor pollicis longus
Certainty: C=1 (Unsure: <67%) C=2 (Mid: >67%) C=3 (Quite sure: >80%)
Feedback
De Quervain's tenosynovitis is one of the most common differential diagnoses of the wrist pain. The
tendons of the 1st dorsal compartment of the wrist (extensor pollicis brevis and abductor pollicis
longus) become inflamed usually related to overuse and less commonly wrist sprain or inflammatory
joint disease.
The correct answer is:
Extensor pollicis brevis and abductor pollicis longus
You did not select a certainty. Assuming: C=1 (Unsure: <67%).
Question 77
Incorrect
CBM mark 0.00
Weight 1.00
Flag question
Question text
You assess a 3-week-old girl in clinic and the parents explain that there is a history of urine leaking
from around the umbilicus. It is associated with local inflammation of the skin. A tentative diagnosis
of urachal fistula is made. You present the history to the paediatric surgical team. What do you
explain to them?
Select one:
The urachus develops from the mesonephric duct and drains into the urogenital sinus
The urachus develops from the mesonephric duct and drains to the allantoic cavity
The urachus develops from the mesonephric duct and drains to the amniotic cavity
The urachus develops from the urogenital sinus and drains to the allantoic cavity
The urachus develops from the urogenital sinus and drains to the amniotic cavity
Certainty: C=1 (Unsure: <67%) C=2 (Mid: >67%) C=3 (Quite sure: >80%)
Feedback
As well as local inflammation, a granuloma may also form.
Gram-positive organisms, such as Staphylococcus aureus and Streptococcus pyogenes, are also
commonly identified.
The correct answer is:
The urachus develops from the urogenital sinus and drains to the allantoic cavity
You did not select a certainty. Assuming: C=1 (Unsure: <67%).
Question 78
Correct
CBM mark 1.00
Weight 1.00
Flag question
Question text
A 64-year-old presents with headaches and a loss of visual acuity. You examine the visual fields and
document that there is loss of visual field represented by the shaded area as shown. Which one of
the following is the most likely cause of the loss of visual field?
Select one:
Left optic nerve lesion
Left optic tract lesion
Pituitary tumour
Right optic nerve lesion
Right optic tract lesion
Certainty: C=1 (Unsure: <67%) C=2 (Mid: >67%) C=3 (Quite sure: >80%)
Feedback
The visual field disturbance shown is a RIGHT homonymous hemianopia, identifying loss of the
RIGHT half of visual field in both eyes.
The Right side of the brain has visual pathways for the LEFT visual fields and vice versa.
The loss of vision shown, indicates a LEFT optic tract lesion. Right optic tract lesion would give the
opposite field loss; pituitary tumour would give a bilateral temporal hemianopia; optic nerve lesions
would give complete loss of vision in the respective eye.
The correct answer is:
Left optic tract lesion
You did not select a certainty. Assuming: C=1 (Unsure: <67%).
Question 79
Correct
CBM mark 1.00
Weight 1.00
Flag question
Question text
A 71-year-old woman is referred having attended the opticians. She has a bitemporal hemianopia.
An MRI scan shows suprasellar extension of a pituitary tumour. Which one of the following is the
most likely explanation for the visual field defect?
Select one:
Bilateral compression of the lateral geniculate bodies
Compression of both optic tracts
Compression of decussating nasal retinal fibres
Compression of the right and left Meyer's Loops
Impaired conduction of afferent impulses from the temporal half of the retinae
Certainty: C=1 (Unsure: <67%) C=2 (Mid: >67%) C=3 (Quite sure: >80%)
Feedback
The nasal retinal fibres decussate in the compressed optic chiasm. Distractors can all cause field
defects.
The correct answer is:
Compression of decussating nasal retinal fibres
You did not select a certainty. Assuming: C=1 (Unsure: <67%).
Question 80
Correct
CBM mark 1.00
Weight 1.00
Flag question
Question text
A 35-year-old construction worker presents with pins and needles of the radial three and a half digits
of her hand that wakes her up at night and is worse after using vibrating machinery. You correctly
diagnose these symptoms are due to a compressive neuropathy in the carpal tunnel. Which of the
following is the correct list of structures that run through the carpal tunnel?
Select one:
Median nerve, 4 tendons of flexor digitorum profundas, four tendons of flexor digitorum superficialis and
flexor carpi radialis tendon
Median nerve, 4 tendons of flexor digitorum profundus, four tendons of flexor digitorum superficialis and
flexor pollicis brevis tendon
Median nerve, 4 tendons of flexor digitorum profundas, four tendons of flexor digitorum superficialis, and
flexor pollicis longus tendon
Ulna nerve, 4 tendons of flexor digitorum profundas, four tendons of flexor digitorum superficialis and
flexor carpi radialis tendon
Ulna nerve, 4 tendons of flexor digitorum profundas, four tendons of flexor digitorum superficialis and
flexor pollicis brevis tendon
Certainty: C=1 (Unsure: <67%) C=2 (Mid: >67%) C=3 (Quite sure: >80%)
Feedback
The carpal tunnel contains the medial nerve which supplies sensation to the radial three and a half
digits consistent with the symptoms stated above. There are 9 tendons that run through the carpal
tunnel, these are correctly stated in option C.
The correct answer is:
Median nerve, 4 tendons of flexor digitorum profundas, four tendons of flexor digitorum superficialis,
and flexor pollicis longus tendon
You did not select a certainty. Assuming: C=1 (Unsure: <67%).

More Related Content

What's hot (20)

Parotidectomy
ParotidectomyParotidectomy
Parotidectomy
 
Neck swellings
Neck swellingsNeck swellings
Neck swellings
 
Collar stud abscess
Collar stud abscessCollar stud abscess
Collar stud abscess
 
Osce surgery
Osce surgeryOsce surgery
Osce surgery
 
Neck swelling
Neck swellingNeck swelling
Neck swelling
 
Ent osce
Ent osceEnt osce
Ent osce
 
Surgical anatomy of thyroid and para thyroid glands. hazem el-folldocx
Surgical anatomy of thyroid and para thyroid glands. hazem el-folldocxSurgical anatomy of thyroid and para thyroid glands. hazem el-folldocx
Surgical anatomy of thyroid and para thyroid glands. hazem el-folldocx
 
PMMC FLAP
PMMC FLAPPMMC FLAP
PMMC FLAP
 
Neck dissection
Neck dissectionNeck dissection
Neck dissection
 
Thyroid surgical anatomy
Thyroid surgical anatomyThyroid surgical anatomy
Thyroid surgical anatomy
 
Inguinal hernia
Inguinal herniaInguinal hernia
Inguinal hernia
 
SURGERY SHORT CASES
SURGERY SHORT CASES SURGERY SHORT CASES
SURGERY SHORT CASES
 
Maxillary sinus carcinoma
Maxillary sinus carcinomaMaxillary sinus carcinoma
Maxillary sinus carcinoma
 
Nasal fracture
Nasal fractureNasal fracture
Nasal fracture
 
VENTRAL HERNIA
VENTRAL HERNIAVENTRAL HERNIA
VENTRAL HERNIA
 
Thyroid Clinical sheet
Thyroid Clinical sheetThyroid Clinical sheet
Thyroid Clinical sheet
 
microtia
microtiamicrotia
microtia
 
Instruments in ent..aak
Instruments in ent..aakInstruments in ent..aak
Instruments in ent..aak
 
NOE fractures
NOE fractures NOE fractures
NOE fractures
 
Parotidectomy
ParotidectomyParotidectomy
Parotidectomy
 

Similar to MRCS Anatomy MCQs .pdf

Adult Chest X-Rays Of The Month - #46
Adult Chest X-Rays Of The Month - #46Adult Chest X-Rays Of The Month - #46
Adult Chest X-Rays Of The Month - #46Sean M. Fox
 
Surgery 209 krok 2
Surgery 209 krok 2Surgery 209 krok 2
Surgery 209 krok 2Raj Twix
 
Oncologcial Emergencies by Prof Ahmed Badheeb 2014 part 1
Oncologcial  Emergencies by Prof Ahmed Badheeb 2014 part 1Oncologcial  Emergencies by Prof Ahmed Badheeb 2014 part 1
Oncologcial Emergencies by Prof Ahmed Badheeb 2014 part 1Prof. Ahmed Mohamed Badheeb
 
Cardiothoracic radiology review
Cardiothoracic radiology reviewCardiothoracic radiology review
Cardiothoracic radiology reviewShivankan Kakkar
 
Drs. Escobar, Pikus, and Blackwell’s CMC X-Ray Mastery Project: January Cases
Drs. Escobar, Pikus, and Blackwell’s CMC X-Ray Mastery Project: January CasesDrs. Escobar, Pikus, and Blackwell’s CMC X-Ray Mastery Project: January Cases
Drs. Escobar, Pikus, and Blackwell’s CMC X-Ray Mastery Project: January CasesSean M. Fox
 
Emergency ct-is it being overused dr.amarnath
Emergency ct-is it being overused dr.amarnathEmergency ct-is it being overused dr.amarnath
Emergency ct-is it being overused dr.amarnathTeleradiology Solutions
 
Heroic procedures you should know
Heroic procedures you should knowHeroic procedures you should know
Heroic procedures you should knowEM OMSB
 
Emergency Quiz Cases
Emergency Quiz CasesEmergency Quiz Cases
Emergency Quiz Casesejheffernan
 
MedReg+1 Bhuva ECGs
MedReg+1 Bhuva ECGsMedReg+1 Bhuva ECGs
MedReg+1 Bhuva ECGsMedReg+1
 
Drs. Lena, Avery, and Davis’s CMC Abdominal Imaging Mastery Project: April Cases
Drs. Lena, Avery, and Davis’s CMC Abdominal Imaging Mastery Project: April CasesDrs. Lena, Avery, and Davis’s CMC Abdominal Imaging Mastery Project: April Cases
Drs. Lena, Avery, and Davis’s CMC Abdominal Imaging Mastery Project: April CasesSean M. Fox
 
Abdominal trauma
Abdominal traumaAbdominal trauma
Abdominal traumawanted1361
 
Spectrum Of Ct Findings In Rupture And Impendinging Rupture Of AAA
Spectrum Of Ct Findings In Rupture And Impendinging Rupture Of AAASpectrum Of Ct Findings In Rupture And Impendinging Rupture Of AAA
Spectrum Of Ct Findings In Rupture And Impendinging Rupture Of AAAXiu Srithammasit
 
Marc Wigoda : Radiotherapy of the Axilla in Early Breast Cancer : When and H...
Marc Wigoda :  Radiotherapy of the Axilla in Early Breast Cancer : When and H...Marc Wigoda :  Radiotherapy of the Axilla in Early Breast Cancer : When and H...
Marc Wigoda : Radiotherapy of the Axilla in Early Breast Cancer : When and H...breastcancerupdatecongress
 
Book 2011 krok 2
Book 2011 krok 2Book 2011 krok 2
Book 2011 krok 2Raj Twix
 
Acute appendicitis in adults - Hoang Cuong HMU
Acute appendicitis in adults - Hoang Cuong HMUAcute appendicitis in adults - Hoang Cuong HMU
Acute appendicitis in adults - Hoang Cuong HMUCường Hoàng
 

Similar to MRCS Anatomy MCQs .pdf (20)

NBME 28 A.pdf
NBME 28 A.pdfNBME 28 A.pdf
NBME 28 A.pdf
 
Adult Chest X-Rays Of The Month - #46
Adult Chest X-Rays Of The Month - #46Adult Chest X-Rays Of The Month - #46
Adult Chest X-Rays Of The Month - #46
 
Surgery 209 krok 2
Surgery 209 krok 2Surgery 209 krok 2
Surgery 209 krok 2
 
Oncologcial Emergencies by Prof Ahmed Badheeb 2014 part 1
Oncologcial  Emergencies by Prof Ahmed Badheeb 2014 part 1Oncologcial  Emergencies by Prof Ahmed Badheeb 2014 part 1
Oncologcial Emergencies by Prof Ahmed Badheeb 2014 part 1
 
Cardiothoracic radiology review
Cardiothoracic radiology reviewCardiothoracic radiology review
Cardiothoracic radiology review
 
Drs. Escobar, Pikus, and Blackwell’s CMC X-Ray Mastery Project: January Cases
Drs. Escobar, Pikus, and Blackwell’s CMC X-Ray Mastery Project: January CasesDrs. Escobar, Pikus, and Blackwell’s CMC X-Ray Mastery Project: January Cases
Drs. Escobar, Pikus, and Blackwell’s CMC X-Ray Mastery Project: January Cases
 
Emergency ct-is it being overused dr.amarnath
Emergency ct-is it being overused dr.amarnathEmergency ct-is it being overused dr.amarnath
Emergency ct-is it being overused dr.amarnath
 
Heroic procedures you should know
Heroic procedures you should knowHeroic procedures you should know
Heroic procedures you should know
 
Emergency Quiz Cases
Emergency Quiz CasesEmergency Quiz Cases
Emergency Quiz Cases
 
MedReg+1 Bhuva ECGs
MedReg+1 Bhuva ECGsMedReg+1 Bhuva ECGs
MedReg+1 Bhuva ECGs
 
Drs. Lena, Avery, and Davis’s CMC Abdominal Imaging Mastery Project: April Cases
Drs. Lena, Avery, and Davis’s CMC Abdominal Imaging Mastery Project: April CasesDrs. Lena, Avery, and Davis’s CMC Abdominal Imaging Mastery Project: April Cases
Drs. Lena, Avery, and Davis’s CMC Abdominal Imaging Mastery Project: April Cases
 
A Case of Pancoast's tumour
A Case of Pancoast's tumourA Case of Pancoast's tumour
A Case of Pancoast's tumour
 
Medulloblastoma
Medulloblastoma Medulloblastoma
Medulloblastoma
 
Abdominal trauma
Abdominal traumaAbdominal trauma
Abdominal trauma
 
Cardiac Metastases from Solid Papillary Carcinoma of the Breast: Analysis of ...
Cardiac Metastases from Solid Papillary Carcinoma of the Breast: Analysis of ...Cardiac Metastases from Solid Papillary Carcinoma of the Breast: Analysis of ...
Cardiac Metastases from Solid Papillary Carcinoma of the Breast: Analysis of ...
 
NBME 28 Q.pdf
NBME 28 Q.pdfNBME 28 Q.pdf
NBME 28 Q.pdf
 
Spectrum Of Ct Findings In Rupture And Impendinging Rupture Of AAA
Spectrum Of Ct Findings In Rupture And Impendinging Rupture Of AAASpectrum Of Ct Findings In Rupture And Impendinging Rupture Of AAA
Spectrum Of Ct Findings In Rupture And Impendinging Rupture Of AAA
 
Marc Wigoda : Radiotherapy of the Axilla in Early Breast Cancer : When and H...
Marc Wigoda :  Radiotherapy of the Axilla in Early Breast Cancer : When and H...Marc Wigoda :  Radiotherapy of the Axilla in Early Breast Cancer : When and H...
Marc Wigoda : Radiotherapy of the Axilla in Early Breast Cancer : When and H...
 
Book 2011 krok 2
Book 2011 krok 2Book 2011 krok 2
Book 2011 krok 2
 
Acute appendicitis in adults - Hoang Cuong HMU
Acute appendicitis in adults - Hoang Cuong HMUAcute appendicitis in adults - Hoang Cuong HMU
Acute appendicitis in adults - Hoang Cuong HMU
 

Recently uploaded

ESC HF 2024 Spotlights Day-2.pptx heart failure
ESC HF 2024 Spotlights Day-2.pptx heart failureESC HF 2024 Spotlights Day-2.pptx heart failure
ESC HF 2024 Spotlights Day-2.pptx heart failuremahiavy26
 
Gross Anatomy and Histology of Tongue by Dr. Rabia Inam Gandapore.pptx
Gross Anatomy and Histology of Tongue by Dr. Rabia Inam Gandapore.pptxGross Anatomy and Histology of Tongue by Dr. Rabia Inam Gandapore.pptx
Gross Anatomy and Histology of Tongue by Dr. Rabia Inam Gandapore.pptxDr. Rabia Inam Gandapore
 
Connective Tissue II - Dr Muhammad Ali Rabbani - Medicose Academics
Connective Tissue II - Dr Muhammad Ali Rabbani - Medicose AcademicsConnective Tissue II - Dr Muhammad Ali Rabbani - Medicose Academics
Connective Tissue II - Dr Muhammad Ali Rabbani - Medicose AcademicsMedicoseAcademics
 
Report Back from SGO: What’s the Latest in Ovarian Cancer?
Report Back from SGO: What’s the Latest in Ovarian Cancer?Report Back from SGO: What’s the Latest in Ovarian Cancer?
Report Back from SGO: What’s the Latest in Ovarian Cancer?bkling
 
CAS 110-63-4 BDO Liquid 1,4-Butanediol 1 4 BDO Warehouse Supply For Excellent...
CAS 110-63-4 BDO Liquid 1,4-Butanediol 1 4 BDO Warehouse Supply For Excellent...CAS 110-63-4 BDO Liquid 1,4-Butanediol 1 4 BDO Warehouse Supply For Excellent...
CAS 110-63-4 BDO Liquid 1,4-Butanediol 1 4 BDO Warehouse Supply For Excellent...ocean4396
 
Quality control tests of suppository ...
Quality control tests  of suppository ...Quality control tests  of suppository ...
Quality control tests of suppository ...Hasnat Tariq
 
Video capsule endoscopy (VCE ) in children
Video capsule endoscopy (VCE ) in childrenVideo capsule endoscopy (VCE ) in children
Video capsule endoscopy (VCE ) in childrenRaju678948
 
Gait deviations in Transtibial prosthesis users
Gait deviations in Transtibial prosthesis usersGait deviations in Transtibial prosthesis users
Gait deviations in Transtibial prosthesis usersJoe Antony
 
VIP Pune 7877925207 WhatsApp: Me All Time Serviℂe Available Day and Night
VIP Pune 7877925207 WhatsApp: Me All Time Serviℂe Available Day and NightVIP Pune 7877925207 WhatsApp: Me All Time Serviℂe Available Day and Night
VIP Pune 7877925207 WhatsApp: Me All Time Serviℂe Available Day and Nightpatanjali9823#S07
 
Sell 5cladba adbb JWH-018 5FADB in stock
Sell 5cladba adbb JWH-018 5FADB in stockSell 5cladba adbb JWH-018 5FADB in stock
Sell 5cladba adbb JWH-018 5FADB in stocktammysayles9
 
^In Pietermaritzburg Hager Werken Embalming +27789155305 Compound Powder in ...
^In Pietermaritzburg  Hager Werken Embalming +27789155305 Compound Powder in ...^In Pietermaritzburg  Hager Werken Embalming +27789155305 Compound Powder in ...
^In Pietermaritzburg Hager Werken Embalming +27789155305 Compound Powder in ...pinkpowder997723
 
The Orbit & its contents by Dr. Rabia I. Gandapore.pptx
The Orbit & its contents by Dr. Rabia I. Gandapore.pptxThe Orbit & its contents by Dr. Rabia I. Gandapore.pptx
The Orbit & its contents by Dr. Rabia I. Gandapore.pptxDr. Rabia Inam Gandapore
 
Renal Replacement Therapy in Acute Kidney Injury -time modality -Dr Ayman Se...
Renal Replacement Therapy in Acute Kidney Injury -time  modality -Dr Ayman Se...Renal Replacement Therapy in Acute Kidney Injury -time  modality -Dr Ayman Se...
Renal Replacement Therapy in Acute Kidney Injury -time modality -Dr Ayman Se...Ayman Seddik
 
Treatment Choices for Slip Disc at Gokuldas Hospital
Treatment Choices for Slip Disc at Gokuldas HospitalTreatment Choices for Slip Disc at Gokuldas Hospital
Treatment Choices for Slip Disc at Gokuldas HospitalGokuldas Hospital
 
Mgr university bsc nursing adult health previous question paper with answers
Mgr university  bsc nursing adult health previous question paper with answersMgr university  bsc nursing adult health previous question paper with answers
Mgr university bsc nursing adult health previous question paper with answersShafnaP5
 
VVIP Hadapsar ℂall Girls 6350482085 Scorching { Pune } Excellent Girl Serviℂe...
VVIP Hadapsar ℂall Girls 6350482085 Scorching { Pune } Excellent Girl Serviℂe...VVIP Hadapsar ℂall Girls 6350482085 Scorching { Pune } Excellent Girl Serviℂe...
VVIP Hadapsar ℂall Girls 6350482085 Scorching { Pune } Excellent Girl Serviℂe...dhyaansingh0898#S07
 
Gallbladder Double-Diverticular: A Case Report المرارة مزدوجة التج: تقرير حالة
Gallbladder Double-Diverticular: A Case Report  المرارة مزدوجة التج: تقرير حالةGallbladder Double-Diverticular: A Case Report  المرارة مزدوجة التج: تقرير حالة
Gallbladder Double-Diverticular: A Case Report المرارة مزدوجة التج: تقرير حالةMohamad محمد Al-Gailani الكيلاني
 
CAD CAM DENTURES IN PROSTHODONTICS : Dental advancements
CAD CAM DENTURES IN PROSTHODONTICS : Dental advancementsCAD CAM DENTURES IN PROSTHODONTICS : Dental advancements
CAD CAM DENTURES IN PROSTHODONTICS : Dental advancementsNaveen Gokul Dr
 
The Clean Living Project Episode 24 - Subconscious
The Clean Living Project Episode 24 - SubconsciousThe Clean Living Project Episode 24 - Subconscious
The Clean Living Project Episode 24 - SubconsciousThe Clean Living Project
 
Sonia Journal club presentation (2).pptx
Sonia Journal club presentation (2).pptxSonia Journal club presentation (2).pptx
Sonia Journal club presentation (2).pptxpalsonia139
 

Recently uploaded (20)

ESC HF 2024 Spotlights Day-2.pptx heart failure
ESC HF 2024 Spotlights Day-2.pptx heart failureESC HF 2024 Spotlights Day-2.pptx heart failure
ESC HF 2024 Spotlights Day-2.pptx heart failure
 
Gross Anatomy and Histology of Tongue by Dr. Rabia Inam Gandapore.pptx
Gross Anatomy and Histology of Tongue by Dr. Rabia Inam Gandapore.pptxGross Anatomy and Histology of Tongue by Dr. Rabia Inam Gandapore.pptx
Gross Anatomy and Histology of Tongue by Dr. Rabia Inam Gandapore.pptx
 
Connective Tissue II - Dr Muhammad Ali Rabbani - Medicose Academics
Connective Tissue II - Dr Muhammad Ali Rabbani - Medicose AcademicsConnective Tissue II - Dr Muhammad Ali Rabbani - Medicose Academics
Connective Tissue II - Dr Muhammad Ali Rabbani - Medicose Academics
 
Report Back from SGO: What’s the Latest in Ovarian Cancer?
Report Back from SGO: What’s the Latest in Ovarian Cancer?Report Back from SGO: What’s the Latest in Ovarian Cancer?
Report Back from SGO: What’s the Latest in Ovarian Cancer?
 
CAS 110-63-4 BDO Liquid 1,4-Butanediol 1 4 BDO Warehouse Supply For Excellent...
CAS 110-63-4 BDO Liquid 1,4-Butanediol 1 4 BDO Warehouse Supply For Excellent...CAS 110-63-4 BDO Liquid 1,4-Butanediol 1 4 BDO Warehouse Supply For Excellent...
CAS 110-63-4 BDO Liquid 1,4-Butanediol 1 4 BDO Warehouse Supply For Excellent...
 
Quality control tests of suppository ...
Quality control tests  of suppository ...Quality control tests  of suppository ...
Quality control tests of suppository ...
 
Video capsule endoscopy (VCE ) in children
Video capsule endoscopy (VCE ) in childrenVideo capsule endoscopy (VCE ) in children
Video capsule endoscopy (VCE ) in children
 
Gait deviations in Transtibial prosthesis users
Gait deviations in Transtibial prosthesis usersGait deviations in Transtibial prosthesis users
Gait deviations in Transtibial prosthesis users
 
VIP Pune 7877925207 WhatsApp: Me All Time Serviℂe Available Day and Night
VIP Pune 7877925207 WhatsApp: Me All Time Serviℂe Available Day and NightVIP Pune 7877925207 WhatsApp: Me All Time Serviℂe Available Day and Night
VIP Pune 7877925207 WhatsApp: Me All Time Serviℂe Available Day and Night
 
Sell 5cladba adbb JWH-018 5FADB in stock
Sell 5cladba adbb JWH-018 5FADB in stockSell 5cladba adbb JWH-018 5FADB in stock
Sell 5cladba adbb JWH-018 5FADB in stock
 
^In Pietermaritzburg Hager Werken Embalming +27789155305 Compound Powder in ...
^In Pietermaritzburg  Hager Werken Embalming +27789155305 Compound Powder in ...^In Pietermaritzburg  Hager Werken Embalming +27789155305 Compound Powder in ...
^In Pietermaritzburg Hager Werken Embalming +27789155305 Compound Powder in ...
 
The Orbit & its contents by Dr. Rabia I. Gandapore.pptx
The Orbit & its contents by Dr. Rabia I. Gandapore.pptxThe Orbit & its contents by Dr. Rabia I. Gandapore.pptx
The Orbit & its contents by Dr. Rabia I. Gandapore.pptx
 
Renal Replacement Therapy in Acute Kidney Injury -time modality -Dr Ayman Se...
Renal Replacement Therapy in Acute Kidney Injury -time  modality -Dr Ayman Se...Renal Replacement Therapy in Acute Kidney Injury -time  modality -Dr Ayman Se...
Renal Replacement Therapy in Acute Kidney Injury -time modality -Dr Ayman Se...
 
Treatment Choices for Slip Disc at Gokuldas Hospital
Treatment Choices for Slip Disc at Gokuldas HospitalTreatment Choices for Slip Disc at Gokuldas Hospital
Treatment Choices for Slip Disc at Gokuldas Hospital
 
Mgr university bsc nursing adult health previous question paper with answers
Mgr university  bsc nursing adult health previous question paper with answersMgr university  bsc nursing adult health previous question paper with answers
Mgr university bsc nursing adult health previous question paper with answers
 
VVIP Hadapsar ℂall Girls 6350482085 Scorching { Pune } Excellent Girl Serviℂe...
VVIP Hadapsar ℂall Girls 6350482085 Scorching { Pune } Excellent Girl Serviℂe...VVIP Hadapsar ℂall Girls 6350482085 Scorching { Pune } Excellent Girl Serviℂe...
VVIP Hadapsar ℂall Girls 6350482085 Scorching { Pune } Excellent Girl Serviℂe...
 
Gallbladder Double-Diverticular: A Case Report المرارة مزدوجة التج: تقرير حالة
Gallbladder Double-Diverticular: A Case Report  المرارة مزدوجة التج: تقرير حالةGallbladder Double-Diverticular: A Case Report  المرارة مزدوجة التج: تقرير حالة
Gallbladder Double-Diverticular: A Case Report المرارة مزدوجة التج: تقرير حالة
 
CAD CAM DENTURES IN PROSTHODONTICS : Dental advancements
CAD CAM DENTURES IN PROSTHODONTICS : Dental advancementsCAD CAM DENTURES IN PROSTHODONTICS : Dental advancements
CAD CAM DENTURES IN PROSTHODONTICS : Dental advancements
 
The Clean Living Project Episode 24 - Subconscious
The Clean Living Project Episode 24 - SubconsciousThe Clean Living Project Episode 24 - Subconscious
The Clean Living Project Episode 24 - Subconscious
 
Sonia Journal club presentation (2).pptx
Sonia Journal club presentation (2).pptxSonia Journal club presentation (2).pptx
Sonia Journal club presentation (2).pptx
 

MRCS Anatomy MCQs .pdf

  • 1. A 3-month-old boy presents to your outpatient clinic as the parents describe seeing clear fluid discharge from the umbilicus. This has been noted since soon after the umbilical cord separated. Which one of the following is the most likely underlying (embryological) explanation? Select one: A patent urachus A patent vitello-intestinal duct A urachal cyst An exomphalos An umbilical granuloma Certainty: C=1 (Unsure: <67%) C=2 (Mid: >67%) C=3 (Quite sure: >80%) Feedback After separation of the cord there should not be any discharge from the umbilicus. The presence of clear fluid points towards this being urine rather than bowel content coming from the umbilicus. A patent urachus implies a persistent collection to the bladder, allowing small amounts of urine to leak but the patient often remains completely well. A patent urachus - correct A patent vitello-intestinal duct - you would expect bowel content, not clear fluid with a connection to the bowel A urachal cyst - a urachal cyst typically will not drain as the urachal tract is obliterated on either side of the cyst An exomphalos - this is an abdominal wall defect with the bowel covered by the cord structures and would be identified soon after birth An umbilical granuloma - a granuloma may leak small amounts of fluid but typically is more serosanguinous The correct answer is: A patent urachus You did not select a certainty. Assuming: C=1 (Unsure: <67%). Question 2 Incorrect CBM mark 0.00 Weight 1.00 Flag question Question text
  • 2. A 40-year-old man presents with clinical and endocrinological features of acromegaly. A microadenoma of the pituitary gland is identified. Which one of the following is the most compelling reason for treating this tumour? Select one: Avoidance of cardiovascular complications Avoidance of colonic cancer Avoidance of diabetes Avoidance of optic pathway compression Avoidance of radiological progression Certainty: C=1 (Unsure: <67%) C=2 (Mid: >67%) C=3 (Quite sure: >80%) Feedback Cardiovascular complications have most effect on survival and are best avoided by early treatment. All other conditions can occur but have less effect upon survival and are therefore less compelling. Radiological progression would initiate additional treatment. Optic pathway compression will not occur with a micro adenoma unless significant growth occurs. The correct answer is: Avoidance of cardiovascular complications You did not select a certainty. Assuming: C=1 (Unsure: <67%). Question 3 Correct CBM mark 1.00 Weight 1.00 Flag question Question text Whilst on call for orthopaedics you diagnose a child with septic arthritis and you are to assist your consultant in theatre performing a washout of the hip joint. The approach used is the anterior approach between the rectus femoris and gluteus medius muscles. Which one of the following inter-nervous planes is used in this approach? Select one: Direct rami of the lumbar plexus and the inferior gluteal nerve
  • 3. Femoral nerve and superior gluteal nerve Lateral cutaneous nerve and inferior gluteal nerve Lateral cutaneous nerve and super gluteal nerve Super gluteal nerve and inferior gluteal nerve Certainty: C=1 (Unsure: <67%) C=2 (Mid: >67%) C=3 (Quite sure: >80%) Feedback The anterior approach is commonly used in paediatrics and exploits the inter-nervous interval between femoral nerve (sartorius and rectus femoris) and superior gluteal nerve ( tensor fascia latae and gluteus medius). The correct answer is: Femoral nerve and superior gluteal nerve You did not select a certainty. Assuming: C=1 (Unsure: <67%). Question 4 Correct CBM mark 1.00 Weight 1.00 Flag question Question text A 1-year-old girl presents with shortness of breath. Her parents have also noticed that her eyes no longer look symmetrical. The right pupil is smaller than the left and the right eyelid appears droopy. A chest X-ray shows increased opacification in the right hemithorax, particularly in the right upper zone. Which one of the following is the most likely diagnosis? Select one: A community acquired pneumonia A congenital diaphragmatic hernia A congenital pulmonary airway malformations Lymphoma Neuroblastoma Certainty: C=1 (Unsure: <67%) C=2 (Mid: >67%) C=3 (Quite sure: >80%) Feedback
  • 4. The clinical features point to a right sided Horner syndrome, secondary to a right apical tumour compressing the sympathetic chain. In this age group the most likely cause is neuroblastoma. A community acquired pneumonia - although a pneumonia can present with shortness of breath, the Horner syndrome is not in keeping A congenital diaphragmatic hernia - although late presenting congenital diaphragmatic hernia can present with shortness of breath, the Horner syndrome is not in keeping A congenital pulmonary airway malformations - although late presenting congenital pulmonary airway malformation can present with shortness of breath (usually because of infection in the CPAM), the Horner syndrome is not in keeping Lymphoma - lymphoma causes lymph node enlargement. The chest X-ray may typically show a mediastinal mass Neuroblastoma - correct The correct answer is: Neuroblastoma You did not select a certainty. Assuming: C=1 (Unsure: <67%). Question 5 Correct CBM mark 1.00 Weight 1.00 Flag question Question text A 25-year-old woman presents with sensory symptoms in the upper limbs and a headache that is worse on coughing. An MRI scan shows a Chiari malformation of the hindbrain and syringomyelia in the cervical spine. Which one of the following nerve tracts are most vulnerable to damage in this condition? Select one: Corticospinal Posterior columns Spinocerebellar Spinothalamic Sympathetic trunks Certainty: C=1 (Unsure: <67%) C=2 (Mid: >67%) C=3 (Quite sure: >80%) Feedback
  • 5. A syrinx expands the centre of the spinal cord. The crossing fibres of the spinothalamic are most at risk of injury. All other fibres are further from the site of pathology. The correct answer is: Spinothalamic You did not select a certainty. Assuming: C=1 (Unsure: <67%). Question 6 Incorrect CBM mark 0.00 Weight 1.00 Flag question Question text An elderly patient with an intra-capsular fractured neck of femur is due to have a hip hemiarthroplasty performed. Which of the following describe anatomical steps to access the hip joint during the most commonly used approach for a hemiarthroplasty? Select one: Detaching the anterior aspect of gluteus medius and minimus from the greater trochanter Detaching piriformis and the short external rotators from their femoral insertion Developing a plane between the gracilis and adductor longus muscles Developing a superficial plane between sartorius and tensor fasciae latae Elevation of vastus lateralis Certainty: C=1 (Unsure: <67%) C=2 (Mid: >67%) C=3 (Quite sure: >80%) Feedback The most commonly used approach for hip hemiarthroplasty is the antero - lateral (modified hardinge) approach to the hip. During this approach the gluteus medius and minimus are detached as one or separate layers from the greater trochanter anteriorly to gain access to the hip joint. The correct answer is: Detaching the anterior aspect of gluteus medius and minimus from the greater trochanter You did not select a certainty. Assuming: C=1 (Unsure: <67%). Question 7 Correct CBM mark 1.00 Weight 1.00
  • 6. Flag question Question text An 80-year-old woman is admitted with a pulsatile mass in the abdomen, guarding and tenderness. An ultrasound demonstrates an intact abdominal aortic aneurysm at the level of the origin of the superior mesenteric artery. You suspect bowel ischaemia of which of the following structures? Select one: The jejunum, ileum and ascending colon The kidneys and spleen The liver and pancreas The stomach and duodenum The transverse and descending colon Certainty: C=1 (Unsure: <67%) C=2 (Mid: >67%) C=3 (Quite sure: >80%) Feedback The celiac trunk supplies the foregut (the liver, stomach and spleen); the SMA supplies the midgut (small intestine and ascending colon) and the IMA supplies the hindgut (transverse, descending and sigmoid colon). The correct answer is: The jejunum, ileum and ascending colon You did not select a certainty. Assuming: C=1 (Unsure: <67%). Question 8 Correct CBM mark 1.00 Weight 1.00 Flag question Question text A 73-year-old man is admitted with severe abdominal pain and guarding. He also has painful tips of the radial 2 digits which are dusky. He is haemodynamically stable but has untreated atrial fibrillation. You suspect an underlying arterioembolic cause and organise an urgent CT angiogram of
  • 7. the bowel, which shows that the inferior mesenteric artery is occluded. Which one of the following will result from the occluded artery? Select one: Ischaemia of the descending colon & sigmoid colon Ischaemia of the duodenum Ischaemia of the jejunum Ischaemia of the jejunum and ileum Ischaemia of the terminal ileum and ascending colon Certainty: C=1 (Unsure: <67%) C=2 (Mid: >67%) C=3 (Quite sure: >80%) Feedback This patient is is suffering with arterial emboli which has led to ischaemia in both the upper limb and bowel. The inferior mesenteric artery supplies the left side of the colon and sigmoid and branches include: left colic ascending branch that joins the middle colic descending branch that joins the highest sigmoid branch sigmoid arteries (2-3) superior sigmoid branch join the left colic inferior sigmoid branch joins the superior rectal The superior mesenteric artery supplies the small bowel and right side of the colon. The correct answer is: Ischaemia of the descending colon & sigmoid colon You did not select a certainty. Assuming: C=1 (Unsure: <67%). Question 9 Correct CBM mark 1.00 Weight 1.00 Flag question Question text A patient who was involved in a road traffic accident suffers a penetrating trauma to the infra- clavicular area and is found to have a pulsatile mass at the level of pectorals minor. You suspect an
  • 8. injury to the axillary artery and the lateral cord of the brachial plexus due to loss of function of which of the following nerves? Select one: Axillary and median nerves Median and musculocutaneous nerves Musculocutaneous and radial nerves Posterior interosseous and anteriorinterosseous nerves Ulnar and median nerves Certainty: C=1 (Unsure: <67%) C=2 (Mid: >67%) C=3 (Quite sure: >80%) Feedback The median and musculocutaneous nerves both arise from the lateral cord of the brachial plexus. The axillary nerve arises from the posterior cord, as does the posterior interosseous branch of the radial nerve. The anterior interosseous is a branch of the median nerve. The correct answer is: Median and musculocutaneous nerves You did not select a certainty. Assuming: C=1 (Unsure: <67%). Question 10 Incorrect CBM mark 0.00 Weight 1.00 Flag question Question text A 9-year-old boy presents with a non-retractile foreskin. He does not currently have any urinary symptoms. On examination the foreskin is thickened, white and stiff. Which one of the following is the most appropriate next step? Select one: A circumcision A preputioplasty Oral steroids
  • 9. Topical betnovate cream Waiting until post puberty to reassess Certainty: C=1 (Unsure: <67%) C=2 (Mid: >67%) C=3 (Quite sure: >80%) Feedback The history points to balanitis xerotica obliterans. This is a progressive scarring condition and requires a circumcision for definitive treatment. A - correct B - this loosens the foreskin but does not remove the disease process C - oral steroids have no place in the management of this localised disease D - Betnovate may control the symptoms for a short period but is not definitive. E - there is no advantage in waiting and there is a small risk of urinary symptoms progressing to the point of urinary retention The correct answer is: A circumcision You did not select a certainty. Assuming: C=1 (Unsure: <67%). The morning following a successful right carotid endarterectomy under general anaesthesia, a 65- year-old man complains of severe hoarseness. Which one of the following is the most likely diagnosis? Select one: The hypoglossal nerve has been damaged during surgery The larynx has been traumatised during anaesthesia The patient has sustained a post operative stroke The recurrent laryngeal nerve has been damaged during surgery There has been bleeding from the endarterectomy site Certainty: C=1 (Unsure: <67%) C=2 (Mid: >67%) C=3 (Quite sure: >80%) Feedback A number of cranial nerves are at risk during carotid surgery and hoarseness secondary to recurrent laryngeal nerve palsy is one of the more common complications. The recurrent laryngeal nerve is at risk during exposure and endarterectomy of the carotid bifurcation due to proximity of the vagus nerve. The correct answer is: The recurrent laryngeal nerve has been damaged during surgery You did not select a certainty. Assuming: C=1 (Unsure: <67%). Question 12
  • 10. Correct CBM mark 1.00 Weight 1.00 Flag question Question text A 10-year-old boy presents to the Emergency Department. Last week he had an upper respiratory tract infection, following which he quickly developed swelling around his right eye and was unable to open it. Now he has bilateral periorbital swelling, a temperature of 40oC and a Glasgow Coma Scale score of 13. He has a contrast enhanced CT scan which demonstrates a cavernous sinus thrombosis. Which one of the following symptoms/signs would not be present? Select one: Absent corneal reflex Anaesthesia over the angle of the mandible Ophthalmoplegia Lateral rectus palsy Absent swinging light pupillary reflex Certainty: C=1 (Unsure: <67%) C=2 (Mid: >67%) C=3 (Quite sure: >80%) Feedback Infected thrombus within the cavernous sinus may cause a variety of cranial nerve palsies affecting those nerves that are directly related to the sinus. Cranial nerves III, IV, V1 and V2 run within the lateral wall of the cavernous sinus. Lesions of CN III in isolation causes the globe to rest in downward, lateral gaze, and interruption of the efferent limb of the pupillary reflexes to light (tested by the swinging light reflex) and accommodation. Lesions of CN IV causes diplopia in downward gaze doe to paralysis of Superior Oblique muscle; while lesions of CN VI cause paralysis of lateral gaze (Lateral Rectus muscle). A combined lesion of CN III, IV and VI causes complete ophthalmoplegia. CN V1, the ophthalmic nerve supplies sensation to the upper face, skull and eye, so a lesion of it affects the afferent limb of the corneal reflex. Sensation to the angle of the mandible is supplied by the mandibular nerve (CN V3), which is not directly related to the cavernous sinus. The correct answer is: Anaesthesia over the angle of the mandible
  • 11. You did not select a certainty. Assuming: C=1 (Unsure: <67%). Question 13 Correct CBM mark 1.00 Weight 1.00 Flag question Question text A 50-year-old man presents with a history of severe low back pain referred down the right leg to the foot. He reports saddle numbness and difficulty with micturition having not had any sensation to pass urine for more than 12 hours. On examination, straight leg raising is limited to 30 degrees on the right and 60 degrees on the left. Motor assessment is difficult due to pain but there appears to be weakness of ankle plantar flexion and eversion. Pinprick and light touch sensation are reduced in the S1 - S4 dermatomes bilaterally. An MRI scan is performed. Which one of the following is the most likely diagnosis? Select one: Discitis Epidural haematoma Prolapsed intervertebral disc S1 root neurofibroma Vertebral haemangioma Certainty: C=1 (Unsure: <67%) C=2 (Mid: >67%) C=3 (Quite sure: >80%) Feedback
  • 12. All pathologies listed occur in this region. The disc has obviously prolapsed on the scan - this fits with the clinical history. The correct answer is: Prolapsed intervertebral disc You did not select a certainty. Assuming: C=1 (Unsure: <67%). Question 14 Correct CBM mark 1.00 Weight 1.00 Flag question Question text A 2-week-old baby has been unable to breast feed since birth and is struggling to bottle feed. The baby cries persistently and according to mum seems to 'burp' and suffer trapped wind. On bottle feeding mother reports regurgitation of milk through the nose. The baby appears underweight and you are concerned about failure to thrive. Which one of the following is the most likely diagnosis? Select one: Branchial cyst Cleft palate Hiatus hernia Pharyngeal pouch Pyloric stenosis Certainty: C=1 (Unsure: <67%) C=2 (Mid: >67%) C=3 (Quite sure: >80%) Feedback Cleft palate causes difficulty in making an adequate seal/suction for breast feeding. Due to excessive air intake bottle feeding is also difficult and results in trapped wind and bloating and milk regurgitation through the nose. Hiatus hernia causes reflux and or vomiting but does not present with breast or bottle feeding issues. Branchial cysts usually present with a midline swelling. Pyloric stenosis presents with projectile vomiting. Pharyngeal pouches present in later life. The correct answer is: Cleft palate
  • 13. You did not select a certainty. Assuming: C=1 (Unsure: <67%). Question 15 Incorrect CBM mark 0.00 Weight 1.00 Flag question Question text A 65-year-old woman develops a hoarse voice, evident in theatre recovery, following an anterior cervical discectomy at the C3/4 level. Anaesthesia was uncomplicated. Surgery, via a 4 cm transverse skin crease incision, was uneventful. A post-operative MRI scan shows no evidence of a haematoma. Which one of the following is the most likely explanation for her symptoms? Select one: External laryngeal nerve transection Laryngeal oedema Recurrent laryngeal nerve transection Superior laryngeal nerve neuropraxis Vagal nerve neuropraxis Certainty: C=1 (Unsure: <67%) C=2 (Mid: >67%) C=3 (Quite sure: >80%) Feedback Recurrent larnygeal not at risk for surgery at this level (much more common with C6/7 pathology). Laryngeal oedema could be a cause but given anaesthetic uncomplicated unlikely. The superior laryngeal is at risk in upper cervical surgery - more commonly due to retraction (neuropraxis) than transection. The correct answer is: Superior laryngeal nerve neuropraxis You did not select a certainty. Assuming: C=1 (Unsure: <67%). Question 16 Correct CBM mark 1.00 Weight 1.00
  • 14. Flag question Question text You see a 56-year-old man in clinic who has presented with left submandibular swelling related to eating. He has had an ultrasound scan which shows a 1.5 cm stone within the substance of the left submandibular gland and you discuss the management options with him, including surgical removal of the gland. Which of the following nerves would you tell him are at risk during this operation? Select one: Accessory, marginal mandibular and lingual nerves Accessory, hypoglossal and lingual nerves Glossopharyngeal, marginal mandibular and lingual nerves Hypoglossal, marginal mandibular and lingual nerves Marginal mandibular, hypoglossal and glossopharyngeal nerves Certainty: C=1 (Unsure: <67%) C=2 (Mid: >67%) C=3 (Quite sure: >80%) Feedback Damage to the hypoglossal nerve causes deviation of the tongue to the affected side. Damage to the marginal mandibular nerve causes weakness of the depressor anguli oris muscle, which pulls the corner of the lower lip downwards. Damage to the lingual nerve causes numbness of the ipsilateral side of the tongue. The correct answer is: Hypoglossal, marginal mandibular and lingual nerves You did not select a certainty. Assuming: C=1 (Unsure: <67%). Question 17 Incorrect CBM mark 0.00 Weight 1.00 Flag question Question text
  • 15. A 32-year-old patient presents to the Emergency Department following a stab injury to the neck. You note a small wound in the anterior triangle and a small pulsatile swelling approximately at the level of the hyoid bone. You also suspect a nerve injury. Which one of the following indicates this? Select one: Failure to 'screw up the eyes' tightly Hearing loss Inability to tense the masseter or temporals muscles Loss of sensation to the upper lip Loss of taste to the anterior 2/3rds of the tongue Certainty: C=1 (Unsure: <67%) C=2 (Mid: >67%) C=3 (Quite sure: >80%) Feedback Many cranial nerves transit the anterior triangle of the neck, namely VII, IX, X, XI and XII. A failure to screw up the eyes indicates an injury to the facial nerve. The other responses indicate damage to nerves that do not transit the anterior triangle; hearing loss would indicate injury to VIII, the anterior 2/3rds of the tongue is supplied by the trigeminal nerve as is sensation to the lip and motor innervation of the masseter and temporals. The correct answer is: Failure to 'screw up the eyes' tightly You did not select a certainty. Assuming: C=1 (Unsure: <67%). Question 18 Incorrect CBM mark 0.00 Weight 1.00 Flag question Question text A pregnant 35-year-old woman is being advised of some potential risks of her new-born child suffering developmental dysplasia of the hip (DDH). Which one of the following is the most significant risk factor associated with DDH? Select one: Breech presentation Family history
  • 16. Female child Firstborn Oligohydramnios Certainty: C=1 (Unsure: <67%) C=2 (Mid: >67%) C=3 (Quite sure: >80%) Feedback The main risk associated with DDH is breech presentation. Female babies are at higher risk than males (ratio of 6:1 female to male). Family history and oligohydramnios are also contributing factors. The correct answer is: Breech presentation You did not select a certainty. Assuming: C=1 (Unsure: <67%). Question 19 Incorrect CBM mark 0.00 Weight 1.00 Flag question Question text A 46-year-old woman presents to the Emergency Department with a headache, diplopia and ocular pain, two weeks following upper respiratory tract infection. She is an insulin dependant diabetic who suffers with stage 2 chronic kidney disease and mild hypertension. Her blood pressure is 130/88, her HBA1C is 84 and her blood glucose levels in the last 14 days have been averaging 16-18mmol/l. Her pulse is 80bpm and regular. Her right eye is pointing slightly down and outwards. Which of the following explains her symptoms best? Select one: Cerebrovascular accident Hemiplegic migraine Intra-cranial hypertension Mono-neuritis Temporal arteritis Certainty: C=1 (Unsure: <67%) C=2 (Mid: >67%) C=3 (Quite sure: >80%) Feedback
  • 17. This is a complicated history with a number of associated features. The finding of note is the downwards and outwards facing eye; indicative of a 3rd cranial nerve (oculomotor) palsy. The aetiology of oculomotor palsy is must-factorial, however, in this history, the patient is not significantly hypertensive and has a regular pulse; unlikely to be a CVA (bleed or embolic). She is however diabetic with significantly raised blood glucose over a number of months (HBA1C). Diabetes is a major cause of mono-neuritis. The correct answer is: Mono-neuritis You did not select a certainty. Assuming: C=1 (Unsure: <67%). Question 20 Incorrect CBM mark 0.00 Weight 1.00 Flag question Question text A 32-year-old man presents to the Emergency Department having come off his mountain bike and hit a tree. He undergoes primary and secondary surveys and the only abnormalities found are bruising over his right mastoid process (Battle's sign) and a complete lower motor neurone facial paralysis. You suspect a temporal bone fracture. Which one of the following examinations or investigations would not be helpful in identifying the level of his facial nerve injury? Select one: Hearing test Pupillary light reflex Salivary flow rate Schirmer's test of lacrimation Stapedial reflex testing Certainty: C=1 (Unsure: <67%) C=2 (Mid: >67%) C=3 (Quite sure: >80%) Feedback Bruising over the mastoid process (Battle's sign) is a sign of a temporal bone fracture. The facial nerve (CN VII) exits the brainstem at the cerebellopontine angle and runs through the temporal bone, initially in close proximity to the vestibulocochlear nerve (CN VIII) in the internal acoustic meatus, and gives off the following branches during its intratemporal course: greater superficial petrosal nerve (parasympathetic fibres to the lacrimal, nasal and palatine glands via the pterygopalatine ganglion), nerve to stapedius, chorda tympani (carrying taste sensation from the
  • 18. anterior 2/3 of the tongue and parasympathetic fibres to the submandibular ganglion). Pupillary light reflex involves the optic (CN II) and oculomotor (CN III) nerves for its afferent and efferent limbs. The correct answer is: Pupillary light reflex You did not select a certainty. Assuming: C=1 (Unsure: <67%). A 5-year-old boy presents to the Emergency Department with a short history of an upper respiratory tract infection that has developed into a left neck swelling, neck stiffness and trismus. He is admitted and treated with intravenous co-amoxiclav, but his symptoms have not improved after 24 hours so you decide to arrange an MRI to exclude a deep neck space infection. The scan shows a collection posterior to the retropharyngeal space, between the alar and prevertebral fascia, and extending from the skull base into the chest. Which one of the following is the lower limit of this space? Select one: It fuses with the diaphragm It fuses with the pericardium T1-2 vertebral level T3-4 vertebral level T5-6 vertebral level Certainty: C=1 (Unsure: <67%) C=2 (Mid: >67%) C=3 (Quite sure: >80%) Feedback The deep spaces of the neck are potential routes for spread of infection. The prevertebral fascia anteriorly splits into the alar and prevertebral fascial layers. The potential space between them contains loose connective tissue and extends from the skull base to the diaphragm. It is often called the 'danger space' since it provides a pathway for infections of the head and neck to spread into the chest, causing mediastinitis and mediastinal abscess. Other tissue spaces in the neck include: Carotid space (extends from skull base to aortic arch); retropharyngeal space (extends from skull base to T1/2); visceral space (extends from hyoid bone and opens into superior mediastinum). The correct answer is: It fuses with the diaphragm You did not select a certainty. Assuming: C=1 (Unsure: <67%). Question 22 Incorrect CBM mark 0.00 Weight 1.00
  • 19. Flag question Question text A 41-year-old man is admitted as an emergency to the plastic surgery ward with cellulitis of the perineal area. It rapidly spreads, resulting in Fournier's gangrene (necrotising fasciitis) and widespread sepsis. This necessitates multiple trips to theatre and admission to ITU. Where is the infection likely to have originated? Select one: Camper fascia Corpora cavernosa and testes External and internal spermatic fascia Scarpa fascia and Colles fascia The inferior fascia of the urogenital diaphragm and Colles fascia Certainty: C=1 (Unsure: <67%) C=2 (Mid: >67%) C=3 (Quite sure: >80%) Feedback The perineal membrane (also known as the inferior fascia of the urogenital diaphragm) and Colles fascia, define an area known as the superficial perineal space. The superficial perineal space contains the membraneous and bulbar urethra and bulbourethral glands. It is adjacent to the anterior anal wall and ischiorectal fossae. Infectious disease of the urethra, bulbourethral glands, perineal structures, or rectum can drain into the superficial perineal space. It may spread into the scrotum or into the anterior abdominal wall and rapidly cephalad via the fascial planes as far as the level of the clavicles. The correct answer is: The inferior fascia of the urogenital diaphragm and Colles fascia You did not select a certainty. Assuming: C=1 (Unsure: <67%). Question 23 Correct CBM mark 1.00 Weight 1.00 Flag question
  • 20. Question text An 80-year-old man who normally walks short distances with a frame and has a past medical history of angina and diabetes, falls and sustains a displaced intracapsular hip fracture. Which of the following is the most appropriate definitive management for him? Select one: Bed rest and traction applied to the injured leg Surgery to perform a dynamic hip screw Surgery to perform a hip hemiarthroplasty Surgery to perform a total hip replacement Surgery to perform open reduction and internal fixation with cannulated screws Certainty: C=1 (Unsure: <67%) C=2 (Mid: >67%) C=3 (Quite sure: >80%) Feedback This is an intracapsular fracture and therefore the blood supply to the head is deemed to be damaged and therefore the most appropriate treatment is to removed the head and perform a hemiarthroplasty. This allows the patient to be mobilised and reduces analgesic requirements. A dynamic hip screw is appropriate for extra capsular fractures where the blood supply to the head is presumed intact. He would not be suitable for a total hip replacement due to his co morbidities and limited mobilisation. Cannulated screws are performed when trying to preserve the femoral head such as in young patients or minimally displaced intracapsular fractures where the blood supply is more likely to remain intact. They my also require the patient to be less than full weight bearing and so are not ideal in elderly people who require frames to walk. Bed rest and traction is reserved for the very few numbers of patients who remain unfit for an anaesthetic despite optimisation. The correct answer is: Surgery to perform a hip hemiarthroplasty You did not select a certainty. Assuming: C=1 (Unsure: <67%). Question 24 Correct CBM mark 1.00 Weight 1.00 Flag question Question text Deviation of the tongue to the right, on attempted protrusion of the tongue, would indicate damage/dysfunction in which of the following cranial nerves?
  • 21. Select one: Left cranial accessory nerve Left hypoglossal nerve Right glossopharyngeal nerve Right hypoglossal nerve Right lingual nerve Certainty: C=1 (Unsure: <67%) C=2 (Mid: >67%) C=3 (Quite sure: >80%) Feedback Each hypoglossal nerve innervates all the extrinsic and intrinsic lingual muscles ipsilaterally except the palatoglossus muscle, the latter being innervated by the vagus via the pharyngeal plexus. The cranial accessory and glossopharyngeal nerves do not innervate any of the lingual muscles and the lingual nerve does not supply any muscles. Deviation of the tongue to the right would indicate paralysis of the right genioglossus muscle and unopposed action of the left genioglossus. The correct answer is: Right hypoglossal nerve You did not select a certainty. Assuming: C=1 (Unsure: <67%). Question 25 Correct CBM mark 1.00 Weight 1.00 Flag question Question text Which extraocular orbital muscle is supplied by the trochlear nerve (IV th cranial nerve)? Select one: Inferior oblique Inferior rectus Medial rectus Superior oblique
  • 22. Superior rectus Certainty: C=1 (Unsure: <67%) C=2 (Mid: >67%) C=3 (Quite sure: >80%) Feedback The superior rectus is supplied by the superior division of the oculomotor nerve while the medial rectus, inferior rectus and inferior oblique are supplied by the inferior division of the oculomotor nerve. The correct answer is: Superior oblique You did not select a certainty. Assuming: C=1 (Unsure: <67%). Question 26 Correct CBM mark 1.00 Weight 1.00 Flag question Question text The superficial 'lobe' of the submandibular gland is separated from the deep 'lobe' by which of the following muscles? Select one: Hyoglossus Mylohyoid Geniohyoid Stylohyoid Genioglossus Certainty: C=1 (Unsure: <67%) C=2 (Mid: >67%) C=3 (Quite sure: >80%) Feedback The submandibular salivary gland wraps itself around the free posterior border of the mylohyoid muscle.The part of the gland that lies superficial to the mylohyoid is termed the superficial 'lobe' while the part that lies deep to the mylohyoid is called the deep 'lobe'. It must be noted that the so-called superficial and deep lobes are always physically continuous with each other around the free posterior border of mylohyoid.
  • 23. The hyoglossus, genioglossus, geniohyoid and stylohyoid muscles lie in a plane altogether deep to the submandibular gland. The correct answer is: Mylohyoid You did not select a certainty. Assuming: C=1 (Unsure: <67%). Question 27 Correct CBM mark 1.00 Weight 1.00 Flag question Question text Which one of the following arteries is a branch of the internal carotid artery? Select one: Infraorbital artery Middle meningeal artery Occipital artery Ophthalmic artery Superficial temporal artery Certainty: C=1 (Unsure: <67%) C=2 (Mid: >67%) C=3 (Quite sure: >80%) Feedback The superficial temporal and occipital arteries are direct branches of the external carotid artery while the middle meningeal and infraorbital arteries are branches of the maxillary artery, in turn a branch of the external carotid artery. The Ophthalmic artery is a direct branch of the internal carotid artery. The correct answer is: Ophthalmic artery You did not select a certainty. Assuming: C=1 (Unsure: <67%). Question 28 Correct CBM mark 1.00 Weight 1.00
  • 24. Flag question Question text A 35-year-old man attends the Emergency Department following a football tackle with a Weber C ankle fracture (high fibula ankle fracture). He requires surgery and the fracture will be fixed with a lag screw and neutralisation plate. Using the direct lateral approach to the fibula which of the following nerves can be potentially injured passing across the fibula from posterior to anterior during your approach? Select one: Deep peroneal nerve Saphenous nerve Superficial peroneal nerve Sural nerve Tibial nerve Certainty: C=1 (Unsure: <67%) C=2 (Mid: >67%) C=3 (Quite sure: >80%) Feedback The superficial peroneal nerve runs from posterior to anterior across the fibula approximately 10cm from the tip of the fibula as it passes distally to supply the skin sensation on the dorsum of the foot. This is the nerve that is most at risk during the approach for this surgery. The correct answer is: Superficial peroneal nerve You did not select a certainty. Assuming: C=1 (Unsure: <67%). Question 29 Correct CBM mark 1.00 Weight 1.00 Flag question Question text
  • 25. A 20-year-old woman underwent a right thoracoscopic sympathectomy for palmar hyperhidrosis. At her follow-up appointment she reports satisfaction with her surgery but volunteers that the right side of her face now feels warm and rather dry. She has also noted some drooping of her right upper eyelid. Which one of the following physical signs is most likely to be present as well? Select one: Constricted right pupil Divergent strabismus Facial hyperaesthesia Facial weakness Normal cilio-spinal reflex Certainty: C=1 (Unsure: <67%) C=2 (Mid: >67%) C=3 (Quite sure: >80%) Feedback Inadvertent damage to the cervical sympathetic chain leads to Horner’s syndrome which is characterised by anhidrosis of the face, ptosis, loss of the cilio-spinal reflex and constriction of the pupil. The pupil becomes constricted due to the unopposed action of the sphincter pupillae muscle, innervated by parasympathetic fibres. In contrast the dilator pupillae muscle is supplied by the cervical sympathetic chain. A functioning dilator pupillae muscle is required for the cilio-spinal reflex. Ptosis is due to weakness of the smooth muscle component of levator palpebrae superioris which is also supplied by the cervical sympathetic chain. Horner syndrome is not associated with a squint or facial weakness as the external ocular and facial muscles do not have motor supply from the sympathetic nervous system. As the cutaneous nerves to the face via branches of the trigeminal nerve are not affected sensation is normal. The correct answer is: Constricted right pupil You did not select a certainty. Assuming: C=1 (Unsure: <67%). Question 30 Correct CBM mark 1.00 Weight 1.00 Flag question Question text A 58-year-old woman presents with a sudden onset of severe headache, associated with nausea and vomiting. On examination her GCS is 13/15. A CT scan is performed (see below). Analgesia and fluids are prescribed. Urgent transfer to the neurosurgical unit is arranged. The neurosurgical registrar advises that nimodipine is commenced. What is the mechanism of action of nimodipine?
  • 26. Select one: ACE inhibitor Alpha blocker Aquaporin Beta Blocker Calcium channel blocker Certainty: C=1 (Unsure: <67%) C=2 (Mid: >67%) C=3 (Quite sure: >80%) Feedback Scan shows SAH. This is communicating rather than obstructive: all ventricles are seen on the scan (lateral, 3rd and 4th) and are dilated. This excludes A, B and D. NPH is a form of communicating hydrocephalus but the presence of the SAH and the raised pressure appearance on the scan (no atrophy) excludes this diagnosis. The correct answer is: Calcium channel blocker You did not select a certainty. Assuming: C=1 (Unsure: <67%). You are assessing a 4-year-old boy in the urology clinic. The boy has recently moved to the UK and has been referred by the General Practitioner. The parents are concerned that the child goes to the toilet many times per day, doesn't like to pass urine standing up instead preferring to sit down, often leaks urine on voiding and has a large tight gathering of foreskin on the top of the penis, but not all the way round. Which one of the following is the most likely treatment? Select one: A mid-stream urine specimen and appropriate treatment for chronic UTI A surgical reconstruction
  • 27. Isolated circumcision Reassurance Referral to a behavioural psychologist Certainty: C=1 (Unsure: <67%) C=2 (Mid: >67%) C=3 (Quite sure: >80%) Feedback Hypospadias affects 1:300 boys and is where the opening of the urethra (meatus) is not at the tip of the penis but either further down or even at the base. The foreskin is gathered at the tip and often on one side only rather than circumferentially. Treatment is surgical, usually at 12 months of age, with reconstruction of the urethra together with circumcision. The correct answer is: A surgical reconstruction You did not select a certainty. Assuming: C=1 (Unsure: <67%). Question 32 Incorrect CBM mark 0.00 Weight 1.00 Flag question Question text You are called to the Emergency Department to review a 2-month-old boy with a 6 hour history of a right groin swelling. The referring doctor says the patient is otherwise well and the abdomen is soft and non tender. The doctor states the mass is in the right inguino-scrotal region, it does not transilluminate but is not reducible. Which one of the following is your preferred management option? Select one: Arrange an urgent ultrasound scan to establish the diagnosis Give analgesia and attempt to reduce the mass Keep the patient nil by mouth, place a nasogastric tube and start broad spectrum antibiotics Schedule the patient for an elective operation in the next few weeks Take the patient to theatre urgently for operative repair Certainty: C=1 (Unsure: <67%) C=2 (Mid: >67%) C=3 (Quite sure: >80%)
  • 28. Feedback The history points to an incarcerated right inguinal hernia. There are no features in the vignette to point to bowel obstruction. In paediatric practice, we would try to reduce the hernia with analgesia (specifically morphine) and assuming this is successful, arrange a semi urgent herniotomy in 48 hours time once the oedema of the tissues in the operative site has settled. A - this is a clinical diagnosis although imaging may occasionally be helpful in atypical cases B - correct C - NBM and NG tube would be indicated in bowel obstruction but attempted hernia reduction takes precedence as it has a high rate of success D - an incarcerated hernia cannot be left untreated as the risk is of bowel ischaemia and perforation. A reducible hernia may be given an elective surgical date. E - immediate surgery is avoided, as long as the hernia can be reduced. This may require consultant input to achieve successful reduction The correct answer is: Give analgesia and attempt to reduce the mass You did not select a certainty. Assuming: C=1 (Unsure: <67%). Question 33 Correct CBM mark 1.00 Weight 1.00 Flag question Question text A 25-year-old motorcyclist involved in an RTC has sustained blunt trauma to the left side of his chest. He is tachypnoeic (RR 20/min) and has an oxygen saturation of 93% on air. He is haemodynamically stable but his chest X-ray shows a large haemothorax. His heart size is normal and the cardiac shadow is not displaced. Oxygen has been provided and intravenous access obtained. In view of the amount of fluid seen in the left pleural cavity an intercostal drain is required. Where is the most appropriate site for insertion of the drain? Select one: 2nd interspace in the mid-clavicular line 2nd intercostal space mid-axillary line 5th interspace between the anterior axillary and mid-axillary lines 5th interspace in the mid-clavicular line
  • 29. 5th interspace posterior to the posterior axillary line Certainty: C=1 (Unsure: <67%) C=2 (Mid: >67%) C=3 (Quite sure: >80%) Feedback The 5th interspace between the anterior axillary and mid-axillary lines is in the “safe area” and is the preferred site for drain insertion. The 5th interspace in the mid-clavicular line normally corresponds to the apex of the left ventricle and therefore drain insertion here risks damage to the heart. Insertion of a drain posterior to the posterior axillary line requires dissection through the latissimus dorsi muscle and a posteriorly sited drain leads to discomfort for the patient when lying down. Insertion of a drain through the 8th interspace risks injury to the diaphragm or even intraabdominal organs. In full expiration the dome of the diaphragm reaches the 4th interspace on the right and the 5th interspace on the left side. The 2nd interspace in the mid-clavicular line is used for urgent needle decompression of a tension pneumothorax. Because of its position a drain at his site would not effectively drain a haemothorax. The correct answer is: 5th interspace between the anterior axillary and mid-axillary lines You did not select a certainty. Assuming: C=1 (Unsure: <67%). Question 34 Correct CBM mark 1.00 Weight 1.00 Flag question Question text A 22 year old male injures his knee while playing football and attends the emergency department. He describes his injury occurring as he pivoted to suddenly change the direction he was running in, it was a non contact injury. He reports immediate swelling in the knee and that he was unable to play on. Which one of the following structures is he most likely to have injured? Select one: Anterior cruciate ligament Lateral Meniscus Medial collateral ligament Medial meniscus Posterior cruciate ligament
  • 30. Certainty: C=1 (Unsure: <67%) C=2 (Mid: >67%) C=3 (Quite sure: >80%) Feedback From this history the most likely injury is an ACL rupture. The mechanism for an ACL rupture is sudden change of direction or pivoting, and is usually a non contact injury common in football and netball players. It produces a large haemarthrosis and therefore is associated with immediate knee swelling and patients not being able to continue the sport or activity they were doing. The correct answer is: Anterior cruciate ligament You did not select a certainty. Assuming: C=1 (Unsure: <67%). Question 35 Correct CBM mark 1.00 Weight 1.00 Flag question Question text A 19-year-old is admitted with a penetrating knife injury. His injuries are consistent with a severed left half of his thoracic spinal cord at the T8 level. Which of the following signs is the most likely consequence of transection of the left dorsal column? Select one: Loss of bladder control Loss of light touch sensation on the left with an upper level 5 cm above the umbilicus Loss of pinprick sensation below the umbilicus on the right Loss of temperature sensation below the umbilicus on the right Loss of vibration sense on the right Certainty: C=1 (Unsure: <67%) C=2 (Mid: >67%) C=3 (Quite sure: >80%) Feedback T8 level approx 5 cm above umbilicus. Dorsal column loss causes ipsilateral loss below level of injury - light touch, conscious proprioception, 2 point discrimination, vibration. Pain (pinprick) and temperature loss are contralateral and start a couple of levels below the injury. They are spinothalamic modalities NOT dorsal column. Bladder control may be lost but it is not a dorsal column function. The correct answer is: Loss of light touch sensation on the left with an upper level 5 cm above the umbilicus
  • 31. You did not select a certainty. Assuming: C=1 (Unsure: <67%). Question 36 Correct CBM mark 1.00 Weight 1.00 Flag question Question text A 41-year-old man presents with a history of right lower limb sciatica. Pain and numbness are reported in the sole and lateral border of the foot. The ankle reflex is absent. At which one of the following levels is the disc prolapse most likely to have occurred? Select one: L2/3 L3/4 L4/5 L5/S1 S1/S2 Certainty: C=1 (Unsure: <67%) C=2 (Mid: >67%) C=3 (Quite sure: >80%) Feedback Compression of the S1 nerve root accounts for the clinical features. The relation of the root to the disc is such that an L5/S1 disc prolapse most commonly compresses the S1 root in isolation as in the scenario. The correct answer is: L5/S1 You did not select a certainty. Assuming: C=1 (Unsure: <67%). Question 37 Incorrect CBM mark 0.00 Weight 1.00 Flag question
  • 32. Question text You are discussing post-operative pain relief during WHO checklist, for a patient about to undergo an open nephrectomy through a loin incision above the 12th rib. Which one of the following methods of post-operative / regional analgesia is most appropriate? Select one: Continual inter-costal infusion Epidural anaesthesia Infra-clavicular block Patient Controlled Analgaesia (PCA) Rectus sheath catheter Certainty: C=1 (Unsure: <67%) C=2 (Mid: >67%) C=3 (Quite sure: >80%) Feedback PCA, epidural and inter-costal infusion are effective post-nephrectomy. Recovery is faster with a local continuous infusion and allows earlier mobilisation and discharge. The rectus sheets should not be entered during a loin approach to the kidney and so a rectus sheath catheter is unhelpful. The correct answer is: Continual inter-costal infusion You did not select a certainty. Assuming: C=1 (Unsure: <67%). Question 38 Incorrect CBM mark 0.00 Weight 1.00 Flag question Question text A 48 hour term, male infant presents with a 12 hour history of green vomiting. The parents state that there have been some wet nappies but that the child has not opened his bowels since birth. The abdomen is distended, full but not tense and not tender. The anus is normally sited. Which one of the following is the most useful diagnostic test? Select one: Blood cultures
  • 33. Lower gastrointestinal contrast study Suction rectal biopsy Ultrasound scan Upper gastrointestinal contrast study Certainty: C=1 (Unsure: <67%) C=2 (Mid: >67%) C=3 (Quite sure: >80%) Feedback The clinical history at this age and with the triad of bile vomiting, abdominal distension and failure to pass meconium points to a likely diagnosis of Hirschsprung's disease. The diagnostic test is therefore a suction rectal biopsy to look for an absence of ganglion cells. The normally sited anus rules out an anorectal anomaly. Suction rectal biopsy - correct Upper gastrointestinal contrast study - the presence of bile vomiting may lead the candidate to consider malrotation and volvulus but typically children with malrotation have passed meconium. If this is felt to be ambiguous then an alternative answer option would be plain abdominal x-ray - this would lead you to upper or lower GI obstruction but not to the underlying cause without further investigation Blood cultures - sepsis is a possible cause but less likely Lower gastrointestinal contrast study - lower GI contrast can be used to investigate lower GI obstruction but will not prove a diagnosis of Hirschsprung's disease Ultrasound scan - USS is unlikely to be contributory here The correct answer is: Blood cultures You did not select a certainty. Assuming: C=1 (Unsure: <67%). Question 39 Correct CBM mark 1.00 Weight 1.00 Flag question Question text You are assessing the neurovascular status in the lower limb of a motorcyclist with an open tibial shaft fracture. There is significant soft tissue loss posteriorly and you are concerned about injury to the posterior tibial artery. You are hopeful that the dorsalis pedis artery is spared as it lies between which one of the following tendons? Select one:
  • 34. Between the extensor hallucis longus and extensor digitorum longus tendons Between the flexor digitorum longus and the flexor halluces longus tendons Between the flexor hallucis longus and flexor digitorum longus tendons Between the tibialis anterior and extensor hallucis longus tendons Between the tibialis posterior and flexor digitorum longus tendons Certainty: C=1 (Unsure: <67%) C=2 (Mid: >67%) C=3 (Quite sure: >80%) Feedback Dorsalis pedis is the continuation of the anterior tibial artery after passing under the extensor retinaculum. It is located on the dorsum of the foot, passing between the tendons of the extensor hallucis longus and extensor digitorum longus. The correct answer is: Between the extensor hallucis longus and extensor digitorum longus tendons You did not select a certainty. Assuming: C=1 (Unsure: <67%). Question 40 Correct CBM mark 1.00 Weight 1.00 Flag question Question text You are asked to assess a patient involved in a road traffic collision who sustained a multi- fragmentary fracture of the distal femur. After considering the displacement forces in this type of injury, which of the following anatomical structure are you most concerned about? Select one: Deep peroneal nerve Popliteal artery Quadriceps tendon Superficial peroneal nerve Tibial nerve
  • 35. Certainty: C=1 (Unsure: <67%) C=2 (Mid: >67%) C=3 (Quite sure: >80%) Feedback Gastrocnemius originates from the supracondylar ridges of the distal femur and inserts via Achilles tendon on the calcaneal tuberosity. In a distal femoral fracture, the heads of this muscle will displace the distal fragment posteriorly, putting at risk the structures of the popliteal fossa. The deepest structure of the popliteal fossa which lies in close proximity to the bone is popliteal artery. The correct answer is: Popliteal artery You did not select a certainty. Assuming: C=1 (Unsure: <67%). You are assessing a patient with an inguinal hernia. The patient has neurological symptoms consistent with meralgia paresthetica as a result of a nerve intrapment into the inguinal ligament. Which of the following is the trapped nerve? Select one: Intermediate cutaneous nerve of the thigh Lateral cutaneous nerve of the thigh Medial cutaneous nerve of the thigh Posterior cutaneous nerve of the thigh Saphenous nerve Certainty: C=1 (Unsure: <67%) C=2 (Mid: >67%) C=3 (Quite sure: >80%) Feedback The lateral cutaneous nerve of the thigh (L2-L3) arises directly from the from the lumbar plexus and usually enters the thigh deep to the inguinal ligament. Occasionally the nerve pierces the inguinal ligament, is compressed here and gives pain and parasthesia over the superior aspect of the outer thigh (meralgia paresthetica). The correct answer is: Lateral cutaneous nerve of the thigh You did not select a certainty. Assuming: C=1 (Unsure: <67%). Question 42 Correct CBM mark 1.00 Weight 1.00 Flag question
  • 36. Question text You are assisting in placing a patient who has suffered a mid-shaft femoral fracture and multiple rib fractures into femoral traction, as he is currently unfit for surgery. You position the traction to balance the displacement of the proximal fragment because of which one of the following? Select one: Adduction by gracilis and flexion by sartorius Flexion and adduction by the adductor magnus Flexion by the iliopsoas muscle and abduction by the gluteus medius and minimus Flexion by the iliopsoas muscle and adduction by the adductors Flexion by the quadriceps and adduction by the semi-membranosus Certainty: C=1 (Unsure: <67%) C=2 (Mid: >67%) C=3 (Quite sure: >80%) Feedback In a proximal femoral fracture the proximal fragment is flexed by the iliopsoas muscle (inserting on the lesser trochanter) and abducted by the hip abductors: gluteus medius and minimus (inserting on the greater trochanter). The distal fragment is adducted by the adductor insertion on the femoral shaft. The correct answer is: Flexion by the iliopsoas muscle and abduction by the gluteus medius and minimus You did not select a certainty. Assuming: C=1 (Unsure: <67%). Question 43 Incorrect CBM mark 0.00 Weight 1.00 Flag question Question text A rock climber slips for 3 meters on a fixed rope and hits the rock-face. Which of the following bones is most likely to have suffered an open fracture? Select one: Femur Humerus
  • 37. Radius Tibia Ulna Certainty: C=1 (Unsure: <67%) C=2 (Mid: >67%) C=3 (Quite sure: >80%) Feedback The tibial shaft is comparatively long, subcutaneous and unprotected anteromedially throughout its entire course. It is also slender in its lower 1/3, making it very vulnerable to fractures in general. Due to lack of a well-developed soft tissue envelope on the anteromedial aspect it is particularly prone to open injuries. The correct answer is: Tibia You did not select a certainty. Assuming: C=1 (Unsure: <67%). Question 44 Correct CBM mark 1.00 Weight 1.00 Flag question Question text Whilst on cardiothoracic placement, you are asked to harvest the long saphenous vein to be used for a coronary artery bypass. How do you locate this structure at the level of the ankle? Select one: Behind the lateral malleolus Behind the medial malleolus Between the tendons of flexor digitorum longus and flexor hallucis longus In front of the lateral mallolus In front of the medial malleolus Certainty: C=1 (Unsure: <67%) C=2 (Mid: >67%) C=3 (Quite sure: >80%) Feedback
  • 38. Correct 'in front of the medial malleolus peroneus longus': Long saphenous vein receives blood from the medial aspect of the foot, passes approximately 2 cm in front of the medial malleolus with saphenous nerve anterior to it and enters the femoral vein in the groin. The correct answer is: In front of the medial malleolus You did not select a certainty. Assuming: C=1 (Unsure: <67%). Question 45 Correct CBM mark 1.00 Weight 1.00 Flag question Question text A 45-year-old man presents with acute lumbar back pain after picking up a heavy box. On examination of his neurology, he has reduced power in dorsiflexion of his left hallux and reduced sensation in the first web space of the left foot. Which of the following is the likely pathology causing these signs and symptoms? Select one: Left sided far lateral disc prolapse at the L3/L4 level Left sided far lateral disc prolaspe at the L4/L5 level Left sided paracentral disc prolapse at the L3/L4 level Left sided paracentral disc prolapse at the L4/L5 level Left sided paracentral disc prolapse at the L5/S1 level Certainty: C=1 (Unsure: <67%) C=2 (Mid: >67%) C=3 (Quite sure: >80%) Feedback The clinical examination shows reduced power and sensation in the L5 nerve root distribution. In the lumbar spine at each disc level there will be a descending and exiting nerve root. The exiting nerve root will be from the level above and the descending nerve root from the level below. For example at the L4/L5 disc level there will be an exiting L4 nerve root and a descending L5 nerve root. Descending nerve roots are affected by paracentral discs and exiting nerve roots by far lateral discs. Therefore with symptoms suggesting L5 nerve root compression the options include a paracentral disc at the L4/L5 level affecting the L5 descending nerve root or a far lateral disc at the level of L5/S1 affecting the exiting L5 nerve root. The correct answer is: Left sided paracentral disc prolapse at the L4/L5 level
  • 39. You did not select a certainty. Assuming: C=1 (Unsure: <67%). Question 46 Correct CBM mark 1.00 Weight 1.00 Flag question Question text A 7-year-old boy presents with a two month history of intermittent peri-umbilical pain and intermittent presence of blood in his faeces. A tentative diagnosis of Meckel diverticulum is made. Which one of the following is true about Meckel diverticulum? Select one: It arises on the mesenteric border of the ileum It is a remnant of the vitelline duct It is a remnant of the vitelline veins It is located 60 cm distal to the ileocaecal valve It is present in 20% of the population Certainty: C=1 (Unsure: <67%) C=2 (Mid: >67%) C=3 (Quite sure: >80%) Feedback Meckel diverticulum is one of the most common congenital digestive system disorders. The embryological origin is thought to be tissue from either the stomach or pancreas. The correct answer is: It is a remnant of the vitelline duct You did not select a certainty. Assuming: C=1 (Unsure: <67%). Question 47 Correct CBM mark 1.00 Weight 1.00 Flag question Question text
  • 40. A patient with recent history of headaches and unsteadiness is found to have a pre-pontine meningioma arising from the clivus. The patient also reports diplopia. Compression of which of the following nerve(s) is most likely to account for the diplopia? Select one: Abducens Oculomotor Ophthalmic division of trigeminal Sympathetic Trochlear Certainty: C=1 (Unsure: <67%) C=2 (Mid: >67%) C=3 (Quite sure: >80%) Feedback A clival tumour is most likely to compress the VI nerve as it ascends from the pontomedullary junction immediately posterior to the clivus. The other nerves are not closely related to the clivus and the ophthalmic division of V and sympathetic do not control eye movements and therefore do not cause diplopia. The correct answer is: Abducens You did not select a certainty. Assuming: C=1 (Unsure: <67%). Question 48 Correct CBM mark 1.00 Weight 1.00 Flag question Question text A 40-year-old amateur soprano singer recently underwent a subtotal thyroidectomy for Graves' disease. When starting back in her choir two weeks after surgery, her fellow members commented that the pitch of her voice had changed. In addition, she noted that she was unable to reach the higher notes that she used to be able to sing quite easily before her operation. Which of the following nerves is most likely to have been injured during her surgery? Select one: External laryngeal nerve Inferior root of the ansa cervicalis
  • 41. Internal laryngeal nerve Recurrent laryngeal nerve Superior root of the ansa cervicalis Certainty: C=1 (Unsure: <67%) C=2 (Mid: >67%) C=3 (Quite sure: >80%) Feedback The external laryngeal nerve which is a branch of the superior laryngeal nerve supplies motor fibres to the crico-thyroid muscle. This muscle tenses the vocal cords and is responsible for the quality and pitch of the voice. In contrast the internal laryngeal nerve, the other branch of the superior laryngeal nerve, is a purely sensory nerve supplying the mucosa of the larynx and pharynx. The recurrent laryngeal nerve supplies the intrinsic muscles of the larynx and injury leads to a either a hoarse voice or even aphonia, if bilateral injury occurs. The ansa cervicalis supplies the following infrahyoid muscles which are depressors of the larynx: sterno-hyoid, omo-hyoid, sterno-thyroid. These muscles are not directly involved in voice production. The correct answer is: External laryngeal nerve You did not select a certainty. Assuming: C=1 (Unsure: <67%). Question 49 Correct CBM mark 1.00 Weight 1.00 Flag question Question text A 55-year-old woman has undergone right mastectomy and level 3 axillary node clearance for a 5 cm node positive, grade 3 invasive ductal carcinoma. The pectoralis minor muscle was divided at its insertion into the coracoid process of the scapula to facilitate removal of enlarged lymph nodes at level 3. Post operation, she complains of numbness and paraesthesia of the medial aspect of her upper arm. Which of the following nerves has been injured during the operation? Select one: Axillary nerve Intercostal-brachial nerve
  • 42. Lateral pectoral nerve Long thoracic nerve Musculo-cutaneous nerve Certainty: C=1 (Unsure: <67%) C=2 (Mid: >67%) C=3 (Quite sure: >80%) Feedback The intercosto-brachial nerve (T2) is the lateral cutaneous branch of the second intercostal nerve. It supplies the skin of the axilla and the skin of the medial aspect of the upper arm close to the axilla. It is at risk of injury during extensive axillary surgery The axillary nerve (C5) supplies the skin of the lateral aspect of the upper arm mainly via the upper lateral cutaneous nerve of the arm. The lateral pectoral nerve (C6,7) is a motor nerve only, supplying the pectoralis major muscle. The long thoracic nerve (C5,6,7) is a motor nerve only supplying the serratus anterior muscle. The musculo-cutaneous nerve (C5,6,7) is a motor nerve to biceps, brachialis and coraco-brachialis and forms the lateral cutaneous nerve of the forearm. The correct answer is: Intercostal-brachial nerve You did not select a certainty. Assuming: C=1 (Unsure: <67%). Question 50 Correct CBM mark 1.00 Weight 1.00 Flag question Question text A 20-year-old man has undergone recent excision biopsy of an enlarged lymph node situated in the posterior triangle of the right side of his neck. The procedure was performed under general anaesthetic. Histology has shown nodular sclerosing Hodgkin's lymphoma. When seen seven days later for the results of the biopsy the patient complains that he has weakness in the region of his right shoulder and he is unable to shrug his shoulder on that side. Injury to which one of the following nerves is likely to account for the patient's symptoms and signs? Select one: Accessory nerve Intermediate supraclavicular nerve
  • 43. Lateral (posterior) supraclavicular nerve Medial supraclavicular nerve Transverse cervical nerve Certainty: C=1 (Unsure: <67%) C=2 (Mid: >67%) C=3 (Quite sure: >80%) Feedback The accessory nerve supplies motor fibres to the trapezius muscle, contraction of which elevates the scapula and the lateral end of the clavicle. The other nerves are all sensory nerves supplying the skin of the neck. The correct answer is: Accessory nerve You did not select a certainty. Assuming: C=1 (Unsure: <67%). A 9-month-old woman, born four weeks prematurely, is presented with a history of poor growth, frequent lethargy and a persistent tachycardia. Auscultation of the chest reveals a systolic murmur, and there is a 'waterhammer' pulse. Blood pressure in the upper and lower limbs are equal and normal. Breathing rate is normal for age. What is the likely diagnosis? Select one: Atrial septal defect Coarctation of the aorta Patent ductus arteriosus Transposition of the great arteries (TGA) Truncus arteriosus Certainty: C=1 (Unsure: <67%) C=2 (Mid: >67%) C=3 (Quite sure: >80%) Feedback Patent ductus arteriosus is associated with systolic murmur and waterhammer pulse. Coarctation of the aorta often produces differential blood pressure between upper and lower limbs. ASD would give a diastolic murmur. TGA presents differently due to reversal of the great arteries leaving the heart. Truncus arteriosus presents much earlier with surgery often required by 3 months of age and breathlessness is often a feature. The correct answer is: Patent ductus arteriosus You did not select a certainty. Assuming: C=1 (Unsure: <67%).
  • 44. Question 52 Correct CBM mark 1.00 Weight 1.00 Flag question Question text An 18-year-old man arrives as a trauma call, he has crashed his motorbike at high speed and hit a bollard head on. He is tachycardic, hypotensive and has bilateral externally rotated legs. Which one of the following pelvic fractures is most likely with the patient's history and presentation? Select one: Acetabular fracture Anterior posterior compression fracture (open book) Fractured neck of femur Lateral compression fracture Vertical sheer fracture Certainty: C=1 (Unsure: <67%) C=2 (Mid: >67%) C=3 (Quite sure: >80%) Feedback The mechanism of injury here is anterior- posterior compression from the petrol tank of the motor bike that would get forced up into the pelvis with a head on collision. These fractures are associated with haemodynamic instability. As the injury involves the pelvis opening up at the front clinically the legs can assume an externally rotated position. A lateral compression and acetabular fracture is associated with impact from one side. The mechanism for a vertical sheer fracture is normally a fall from height, although it is often associated with haemodynamic instability. The correct answer is: Anterior posterior compression fracture (open book) You did not select a certainty. Assuming: C=1 (Unsure: <67%). Question 53 Incorrect CBM mark 0.00 Weight 1.00
  • 45. Flag question Question text You are called to the post-natal ward to see a 5-day-old girl who was born 14 days prematurely. The paediatric team had noticed an erythematous inflamed area between the genitalia and the anus, with evidence of localised inflammation. The fluid chart has documented very loose stools, green/brown in colour. The baby is otherwise feeding normally. Which one of the following is the most likely diagnosis? Select one: Fistula Hirschsprung's disease Hypospadias Perineal groove Prolapsed anus Certainty: C=1 (Unsure: <67%) C=2 (Mid: >67%) C=3 (Quite sure: >80%) Feedback Perineal groove is a rare anomaly that occurs primarily in female infants. It is thought to be either a failure of midline fusion or a urorectal septum developmental defect during cloacal embryological stages at 5th to 8th week of gestation. In many cases the area self-epithelialise, though observation is needed to prevent infection. Hirschsprung's presents with constipation or failure to pass faeces. A fistula should form part of your differential, but would be an obvious communication leaking bowel content. Hypospadias occurs in male infants. The correct answer is: Perineal groove You did not select a certainty. Assuming: C=1 (Unsure: <67%). Question 54 Correct CBM mark 1.00 Weight 1.00
  • 46. Flag question Question text You assess an elderly patient who has been diagnosed with a pharyngeal pouch. This is an out pouching of the oesophagus through an area called Killian dehiscence between which muscles of the pharynx? Select one: Inferior constrictor and cricopharyngeus Salpingopharyngeus and inferior constrictor Salpingopharyngeus and superior constrictor Superior and inferior constrictors Superior constrictor and cricopharyngeus Certainty: C=1 (Unsure: <67%) C=2 (Mid: >67%) C=3 (Quite sure: >80%) Feedback Killian dehiscence is between the inferior constrictor and cricopharynxgeus. Aetiology of pharyngeal pouch is thought to be related to dysfunction of the cricopharyngeus. The correct answer is: Inferior constrictor and cricopharyngeus You did not select a certainty. Assuming: C=1 (Unsure: <67%). Question 55 Incorrect CBM mark 0.00 Weight 1.00 Flag question Question text A 60-year-old woman undergoes surgery to clip a posterior inferior cerebellar artery aneurysm. Cranial nerve injury may occur during the surgical exposure. Which one of the following deficits is most commonly associated with this operation post-operatively?
  • 47. Select one: Diplopia Facial weakness and numbness Hoarse voice/weak cough Tongue atrophy Visual field defect Certainty: C=1 (Unsure: <67%) C=2 (Mid: >67%) C=3 (Quite sure: >80%) Feedback The cranial nerves intimately related to the PICA vessel and therefore vulnerable to injury during this procedure are IX and X. This causes hoarse voice/weak cough The correct answer is: Hoarse voice/weak cough You did not select a certainty. Assuming: C=1 (Unsure: <67%). Question 56 Correct CBM mark 1.00 Weight 1.00 Flag question Question text A 50-year-old-woman presents to the upper GI clinic with a six month history of sporadic central abdominal pain. She also suffers mild dyspepsia. She has been a lifetime heavy smoker and over the last 18 months has had investigations for peripheral vascular disease. Her pain only arises after eating, especially larger meals. Which one of the following is the most likely cause for her symptoms? Select one: Barrets oesophagus Biliary cholic Irritable bowel disease Oesophageal spasm
  • 48. Superior mesenteric stenosis Certainty: C=1 (Unsure: <67%) C=2 (Mid: >67%) C=3 (Quite sure: >80%) Feedback The history of post-prandial abdominal pain is indicative of vascular insufficiency to the gut, especially given the history of smoking and recent peripheral vascular disease. The correct answer is: Superior mesenteric stenosis You did not select a certainty. Assuming: C=1 (Unsure: <67%). Question 57 Correct CBM mark 1.00 Weight 1.00 Flag question Question text A 32-year-old patient is undergoing an emergency laparotomy following blunt trauma to the abdomen. On opening the peritoneum there is torrential bleeding and your consultant performs 'Pringle's manoeuvre'. Which one of the following structures are clamped by this manoeuvre? Select one: Gastroduodenal artery and celiac trunk Hepatic artery and portal vein Inferior vena cava and portal vein Splenic artery and portal vein Superior vena cava and hepatic artery Certainty: C=1 (Unsure: <67%) C=2 (Mid: >67%) C=3 (Quite sure: >80%) Feedback First described by J. Pringle, this 'manoeuvre' essentially prevents inflow of blood to the liver by compressing the hepatic artery and portal vein. Performed either manually or with a clamp, it helps in controlling hepatic haemorrhage. The common bile duct is also compressed by this action. The correct answer is: Hepatic artery and portal vein
  • 49. You did not select a certainty. Assuming: C=1 (Unsure: <67%). Question 58 Incorrect CBM mark 0.00 Weight 1.00 Flag question Question text A 25-years-old motorcyclist has been admitted to the Emergency Department following a road traffic collision. His only significant injury is that of a probable fracture of the right humerus which is confirmed by X-ray showing a displaced fracture of the mid shaft of the humerus. He complains of weakness in the right arm and hand and appears to have a wrist drop. Which of the following combinations of movements is most likely to be impaired? Select one: Extension of the elbow and wrist joints Extension of the elbow, wrist and metacarpo-phalangeal joints Extension of the metacarpo-phalangeal and interphalangeal joints Extension of the wrist and metacarpo-phalangeal joints Extension of the wrist, metacarpo-phalangeal and interphalangeal joints Certainty: C=1 (Unsure: <67%) C=2 (Mid: >67%) C=3 (Quite sure: >80%) Feedback The patient has suffered injury to the radial nerve where it courses through the spiral groove of the humerus. The nerve supply to the triceps muscle from the radial nerve is at a more proximal level and so elbow extension is preserved. The extensor muscles of the wrist joint such as extensor carpi radialis (longus and brevis) and extensor carpi ulnaris will be affected by the injury. Similarly, the muscles extending the metacarpo-phalangeal joints joints such as extensor digitorum will be weakened. However, extension of the interphalangeal joints can still occur due the action of the lumbrical muscles which are supplied buy the ulnar and median nerves. The correct answer is: Extension of the wrist and metacarpo-phalangeal joints You did not select a certainty. Assuming: C=1 (Unsure: <67%). Question 59 Incorrect CBM mark 0.00 Weight 1.00
  • 50. Flag question Question text A 12-year-old girl presents with a 2 cm diameter anterior midline cyst of the neck. It moves on swallowing and on protrusion of her tongue. At operation the cyst is very closely associated with a bone derived from two of the embryological pharyngeal arches. From which one of the following pharyngeal arches is this bone derived? Select one: 1st and 2nd arches 2nd and 3rd arches 3rd and 4th arches 4th and 5th arches 5th and 6th arches Certainty: C=1 (Unsure: <67%) C=2 (Mid: >67%) C=3 (Quite sure: >80%) Feedback The clinical features are typical of a thyroglossal cyst which is derived from remnants of the thyroglossal duct. Embryologically this duct passes close to the anterior part of the body of the hyoid bone, often also passing posterior to the bone before descending towards the thyroid gland. The hyoid bone is derived from the 2nd and 3rd pharyngeal arches. The 1st arch forms the maxilla and mandible. The 4th and 6th arches form the muscles and cartilage of the larynx whilst the 5th arch usually disappears. The correct answer is: 2nd and 3rd arches You did not select a certainty. Assuming: C=1 (Unsure: <67%). Question 60 Incorrect CBM mark 0.00 Weight 1.00 Flag question Question text
  • 51. You see a 34-year-old woman in the Emergency Department following a road traffic collision. The ambulance crew reported a Glasgow Coma Scale (GCS) of 9 with pupils that were equally reactive to light. You record her GCS as 5 and note that she now has a fixed dilated pupil on the left. A CT scan is performed. Which one of the following is the clinically most urgent finding on this scan? Select one: Acute subdural haematoma Extradural haematoma Skull base fracture Temporal lobe contusion Traumatic subarachnoid haemorrhage Certainty: C=1 (Unsure: <67%) C=2 (Mid: >67%) C=3 (Quite sure: >80%) Feedback All 5 of the options are shown on this trauma scan. The obvious abnormality is the mass on the left side (using standard radiology convention for side). This has the classic lens shape. Although not particularly large it accounts for the dilated pupil on the left and warrants emergency surgery. The correct answer is: Extradural haematoma
  • 52. You did not select a certainty. Assuming: C=1 (Unsure: <67%). An 8- year-old man presents to clinic via the 2-week wait pathway with a complete right lower motor neurone facial paralysis and a painless mass in his ipsilateral parotid gland. Which one of the following is the most likely histological diagnosis? Select one: Acinic cell carcinoma Adenoid cystic carcinoma Carcinoma ex-pleomorphic adenoma Lymphoma Squamous cell carcinoma Certainty: C=1 (Unsure: <67%) C=2 (Mid: >67%) C=3 (Quite sure: >80%) Feedback 25% of salivary gland malignancy presents in the parotid gland (50% in the submandibular and 75% in minor salivary glands). The presence of a parotid mass mass with a facial nerve paralysis is highly suspicious for malignancy. The commonest malignant parotid tumour is adenoid cystic carcinoma (approx. 25%), and poorly differentiated carcinoma (approx. 25%), then carcinoma ex-pleomorphic (approx. 10%), then acinic cell carcinoma (3%). The correct answer is: Adenoid cystic carcinoma You did not select a certainty. Assuming: C=1 (Unsure: <67%). Question 62 Correct CBM mark 1.00 Weight 1.00 Flag question Question text A 54-year-old man returns from a ski holiday. Following a fall onto the outstretched hand he developed pain lasting about 10 minutes in his 3rd digit (middle finger). He immediately noticed that his distal phalanx was flexed at the DIP joint. He was unable to voluntarily extend the DIP, although he could passively straighten the finger without difficulty. An X-ray showed no evidence of a fracture. What is the name of this injury? Select one:
  • 53. Boutonniere deformity Hammer finger Mallet finger Swan neck deformity Trigger finger Certainty: C=1 (Unsure: <67%) C=2 (Mid: >67%) C=3 (Quite sure: >80%) Feedback This is a classical description of a mallet finger. The distal attachment of the extensor tendon is avulsed from the terminal phalanx. Distractors all affect the fingers but are incorrect. The correct answer is: Mallet finger You did not select a certainty. Assuming: C=1 (Unsure: <67%). Question 63 Incorrect CBM mark 0.00 Weight 1.00 Flag question Question text A 25-year-old woman falls from a horse and sustains a cervical spine injury. On arrival at the Emergency Department, she is assessed and stabilised as per the ATLS principles. During the secondary surgery examination she has 5/5 power in elbow flexion and 0/5 power below this level bilaterally. Sensation is normal on the lateral aspect of the arm over the deltoid region down to the elbow but is abnormal from the elbow down to the hand bilaterally. Perianal sensation is intact. How would you describe the spinal cord injury? Select one: C4 Complete injury C4 Incomplete injury C5 Complete injury C5 Incomplete injury
  • 54. C6 Incomplete injury Certainty: C=1 (Unsure: <67%) C=2 (Mid: >67%) C=3 (Quite sure: >80%) Feedback To determine the single neurological level you must work out the lowest segment where motor and sensory function is normal bilaterally. This patient has normal elbow flexion (C5) and sensation over the lateral aspect of the arm down to the elbow (C5) and so the sensory and motor level is C5. Perianal sensation is preserved and so the injury is incomplete. The correct answer is: C5 Incomplete injury You did not select a certainty. Assuming: C=1 (Unsure: <67%). Question 64 Correct CBM mark 1.00 Weight 1.00 Flag question Question text A 25-year-old motorcyclist is admitted following a high speed road traffic collision. He is paraplegic with a sensory level at T5. He is noted to have priapism. Which one of the following is the most likely explanation of the priapism? Select one: Excitation of the sympathetic trunk Loss of inhibition of the parasympathetic nervi erigentes Loss of inhibition of the sympathetic output Pudendal nerve lesion Sacral root lesion Certainty: C=1 (Unsure: <67%) C=2 (Mid: >67%) C=3 (Quite sure: >80%) Feedback Priapism is due to loss of descending inhibition of sacral parasympathetic output. Other distractors are reasonable considerations but all are incorrect. The correct answer is: Loss of inhibition of the parasympathetic nervi erigentes You did not select a certainty. Assuming: C=1 (Unsure: <67%).
  • 55. Question 65 Incorrect CBM mark 0.00 Weight 1.00 Flag question Question text A 68-year-old woman presents with symptoms consistent with a prolapsed cervical disc. She reports pain and numbness in the right lateral forearm, thenar eminence and thumb. The biceps and supinator reflexes are absent. At which level is the prolapsed disc most likely to have occurred? Select one: C3/4 C4/5 C5/6 C6/7 C7/T1 Certainty: C=1 (Unsure: <67%) C=2 (Mid: >67%) C=3 (Quite sure: >80%) Feedback The description of symptoms is characteristic of C6 root compression. This is most commonly compressed by a prolapsed C5/6 disc due to the relationship between nerve root and the disc. The correct answer is: C5/6 You did not select a certainty. Assuming: C=1 (Unsure: <67%). Question 66 Correct CBM mark 1.00 Weight 1.00 Flag question Question text
  • 56. A 65-year-old farmer presents to the Emergency Department after losing control of a circular saw and sustaining a laceration posterior to the medial malleolus. You examine the wound under local anaesthetic to identify if any structures have been damaged. When working from anterior to posterior, which is the correct order of structures passing behind the medial malleolus? Select one: Extensor digitorum longus tendon, posterior tibial artery, tibial nerve, extensor hallucis longus tendon and tibialis posterior tendon Flexor digitorum longus tendon, posterior tibial artery, tibial nerve, flexor hallucis longus tendon and tibialis posterior tendon Flexor digitorum longus tendon, tibialis posterior tendon, posterior tibial artery, tibial nerve and flexor hallucis longus tendon Tibialis posterior tendon, flexor digitorum longus tendon, posterior tibial artery, tibial nerve and flexor hallucis longus tendon Tibialis posterior tendon, posterior tibial artery, tibial nerve, flexor digitorum longus tendon and flexor hallucis longus tendon Certainty: C=1 (Unsure: <67%) C=2 (Mid: >67%) C=3 (Quite sure: >80%) Feedback The correct order is option D, when considering the structures responsible for flexing/ extending the toes, flexors run behind the medial malleolus and the extensors pass anterior to the ankle joint. The correct answer is: Tibialis posterior tendon, flexor digitorum longus tendon, posterior tibial artery, tibial nerve and flexor hallucis longus tendon You did not select a certainty. Assuming: C=1 (Unsure: <67%). Question 67 Incorrect CBM mark 0.00 Weight 1.00 Flag question Question text A 4-year-old boy sustains a closed extension-type supracondylar fracture of the humerus. Which one of the following nerves is most likely to suffer a neuropraxia? Select one: Anterior interosseous nerve
  • 57. Axillary nerve Posterior interosseous nerve Radial nerve Ulna nerve Certainty: C=1 (Unsure: <67%) C=2 (Mid: >67%) C=3 (Quite sure: >80%) Feedback The anterior interosseous nerve is most commonly injured in extension type injuries. Posterior interosseous injury is usually associated with radial head fractures. Ulnar neuropraxia (not listed) is associated with flexion-type injuries; ulna nerve direct injury is commonly associated with surgical fine-wire insertion. The correct answer is: Anterior interosseous nerve You did not select a certainty. Assuming: C=1 (Unsure: <67%). Question 68 Correct CBM mark 1.00 Weight 1.00 Flag question Question text A sciatic nerve block is performed just above the popliteal fossa for a patient who is about to undergo ankle fracture fixation under general anaesthetic. Which one of the following areas is likely to have normal sensation post-operatively? Select one: 1st dorsal webspace Anterolateral aspect of the foot Dorsomedial aspect of the foot Lesser toes Sole of the foot
  • 58. Certainty: C=1 (Unsure: <67%) C=2 (Mid: >67%) C=3 (Quite sure: >80%) Feedback The block will affect branches of the sciatic nerve, but will not affect the terminal branch of the femoral nerve, namely the saphenous distribution providing sensation along the medial border of the foot. The correct answer is: Dorsomedial aspect of the foot You did not select a certainty. Assuming: C=1 (Unsure: <67%). Question 69 Incorrect CBM mark 0.00 Weight 1.00 Flag question Question text You have examined a 12-month-old child in paediatric surgical clinic with a diagnosis of syndactaly of the fingers. You are explaining the diagnosis and history to the parents. Which of the following is correct? Select one: Occurs in 1:1,000, caucasian male preponderance, sporadic inheritance pattern, usually unilateral, commonly affects the 3rd webspace Occurs in 1:3,000, asian female preponderance, autosomal recessive inheritance, usually unilateral, commonly affects the 1st webspace Occurs in 1:3,000, caucasian male preponderance, autosomal dominant inheritance, often bilateral, commonly affects the 3rd webspace Occurs in 1:3,000, caucasian male preponderance, autosomal dominant inheritance, usually unilateral, commonly affects the 1st webspace Occurs in 1:30,000, asian female preponderance, autosomal dominant inheritance, often bilateral, commonly affects the 3rd webspace Certainty: C=1 (Unsure: <67%) C=2 (Mid: >67%) C=3 (Quite sure: >80%) Feedback Occurs in 1:3,000 More common in caucasians rather than black or asian.
  • 59. More common in males. Autosomal dominant inheritance, often bilateral. Commonly affects the 3rd webspace, rare in the first The correct answer is: Occurs in 1:3,000, caucasian male preponderance, autosomal dominant inheritance, often bilateral, commonly affects the 3rd webspace You did not select a certainty. Assuming: C=1 (Unsure: <67%). Question 70 Correct CBM mark 1.00 Weight 1.00 Flag question Question text A 26-year-old woman presents with a cough headache, tingling in the hands, and loss of pinprick and temperature sensation in the hands. An MRI scan shows downward herniation of the cerebellar tonsils and mid-cervical syringomyelia. Which of the following tracts is most likely to malfunction as a result of this pathology? Select one: Corticospinal Dorsal columns Spinocerebellar Spinothalamic Vestibulospinal Certainty: C=1 (Unsure: <67%) C=2 (Mid: >67%) C=3 (Quite sure: >80%) Feedback Herniation of tonsils compresses the craniocervical junction and cases syringomyelia. The spinothalamic tracts cross in region of central canal - these are vulnerable to damage when a syrinx expands. Typically causes tingling, burns on hands in a "suspended" or cape-like distribution. The correct answer is: Spinothalamic You did not select a certainty. Assuming: C=1 (Unsure: <67%).
  • 60. A patient suffers injury to the common peroneal nerve following a high fibular fracture. After six months, recovery has been minimal and she complains of persistent foot-drop. Having failed conservative measures, which one of the following tendon transfer procedures would be appropriate? Select one: Extensor digitorum longus Extensor hallucis longus Peroneus brevis Peroneus longus Tibialis posterior Certainty: C=1 (Unsure: <67%) C=2 (Mid: >67%) C=3 (Quite sure: >80%) Feedback The only tendon unaffected by a common peroneal nerve injury is the tibialis posterior tendon. Despite being a plantar flexor invertor, it can be transferred into the anterior compartment to act as an ankle dorsiflexor. The correct answer is: Tibialis posterior You did not select a certainty. Assuming: C=1 (Unsure: <67%). Question 72 Correct CBM mark 1.00 Weight 1.00 Flag question Question text An 18-year-old female motorcyclist is involved in a high-speed road traffic accident. Primary survey reveals a patient in respiratory distress with tracheal deviation towards the left. Breath sounds in the right side of the chest are reduced. What is the most appropriate next step in assessment and treatment of this patient? Select one: Left sided tube intercostal thoracostamy Needle pericardiocentesis
  • 61. Right sided needle decompression thoracostomy Urgent chest radiograph Urgent CT with possible interventional radiology Certainty: C=1 (Unsure: <67%) C=2 (Mid: >67%) C=3 (Quite sure: >80%) Feedback This is a RIGHT sided tension pneumothorax; the breath sounds are reduced on the right and the trachea is being pushed towards the left. The right side of the chest requires de-tensioning. Decompression of the left side will worsen the clinical situation. The correct answer is: Right sided needle decompression thoracostomy You did not select a certainty. Assuming: C=1 (Unsure: <67%). Question 73 Correct CBM mark 1.00 Weight 1.00 Flag question Question text A 75-year-old woman attends to have a left shoulder replacement. A delto-pectoral approach is the approach the surgeon will be using and is a true inter-nervous plane. Which one of the following is the inter-nervous plane? Select one: Axillary nerve and medial and lateral pectoral nerves Axillary nerve and medial pectoral nerve Lateral pectoral nerve and musculotanous nerve Musculotanous nerve and radial nerve Radial nerve and Axillary nerve Certainty: C=1 (Unsure: <67%) C=2 (Mid: >67%) C=3 (Quite sure: >80%) Feedback
  • 62. The delto-pectoral approach is between the deltoid muscle and the pectoralis major muscle. The nerve supplying the deltoid is the axillary nerve and the nerves supplying the pectoralis major muscle are the medial an lateral pectoral nerves. Pectoralis minor is supplied just by the medial pectoral nerve. The correct answer is: Axillary nerve and medial and lateral pectoral nerves You did not select a certainty. Assuming: C=1 (Unsure: <67%). Question 74 Incorrect CBM mark 0.00 Weight 1.00 Flag question Question text You see a 34-year-old man in the Emergency Department following a road traffic collision. The ambulance crew reported a Glasgow Coma Score (GCS) of 9 with pupils that were equally reactive to light. You record his GCS as 5 and note that he now has a fixed dilated pupil on the right. Which one of the following is the most likely explanation for the pupillary dilatation? Select one: Compression of abducens nerve Direct compression of the midbrain Disruption of ascending sympathetic fibres caused by a carotid artery Ischaemia of the Edinger-Westphal nucleus Loss of parasympathetic innervation of the pupil Certainty: C=1 (Unsure: <67%) C=2 (Mid: >67%) C=3 (Quite sure: >80%) Feedback The pupil is innervated by autonomic nerve fibres. Parasympathetic fibres cause pupillary constriction, travelling from the Edinger–Westphal nuclei in the midbrain via cranial nerve III to the pupilloconstrictor muscle. Increased intracranial pressure may cause compromise of this pupillary response and is one of the few signs detectable in the unconscious patient. The correct answer is: Loss of parasympathetic innervation of the pupil You did not select a certainty. Assuming: C=1 (Unsure: <67%). Question 75
  • 63. Incorrect CBM mark 0.00 Weight 1.00 Flag question Question text A 25-year-old motorcyclist presents to the Emergency Department at 23:00 hours with an isolated high energy open fracture of the left tibia. He was wearing full protective clothing and was not in an agricultural or aquatic environment. On examination he has a 7 cm laceration overlying the subcutaneous border of the tibia which is visible and there is soft tissue loss. His pulses are intact distally and radiographs show a spiral fracture of the tibial mid shaft. IV antibiotics have been given and tetanus status checked. Which one of the following is the most appropriate management? Select one: Apply a saline soaked gauze and occlusive dressing to the wound and splint the limb, after manipulation if indicated. Aim for theatre for a joint orthoplastics procedure at the next available opportunity within 12 hours. Prepare the patient to be taken to theatre immediately for lower limb fasciotomies to decompress compartments as this injury is high risk for compartment syndrome. Prepare the patient to be taken to theatre immediately for washout and debridement of the wound and application of an external fixator by the orthopaedic team. Wash the wound in ED with sterile saline and dress it with a betadine soaked gauze. Aim for theatre for a joint orthoplastics procedure at the next available opportunity within 12 hours. Washout the wound in ED with sterile saline and dress with a betadine soaked gauze. Splint the limb, after manipulation if indicated. Aim for theatre for a joint orthoplastics procedure within the next 24 hours. Certainty: C=1 (Unsure: <67%) C=2 (Mid: >67%) C=3 (Quite sure: >80%) Feedback The British Orthopaedic Association and British Association of Plastic Reconstructive and Aesthetic Surgeons joint guideline for open fractures states that intravenous antibiotics should be administered ideally within 1 hour of injury. Wounds should only be handled to remove gross contamination, dressed with saline soaked gauze and covered with an occlusive film. Washouts in the emergency department are not indicated, nor is betadine soaked dressings. Indications for immediate theatre intervention include highly contaminated wounds (agricultural, aquatic, sewage), arterial injuries and signs of compartment syndrome. For solitary high energy open fractures, the aim is to be in theatre within 12 hours of injury. Initial surgery should be carried out by consultants in orthopaedics and plastic surgery - a combined orthoplastic approach. The correct answer is:
  • 64. Apply a saline soaked gauze and occlusive dressing to the wound and splint the limb, after manipulation if indicated. Aim for theatre for a joint orthoplastics procedure at the next available opportunity within 12 hours. You did not select a certainty. Assuming: C=1 (Unsure: <67%). Question 76 Correct CBM mark 1.00 Weight 1.00 Flag question Question text You are asked to review a 50-year-old medical secretary with progressive, radial sided wrist pain. On examination, the pain is worse on ulnar deviation yet range of movement of the wrist joint is good. Radiographs were unremarkable. You suspect a De Quervain's tenosynovitis because of inflammation in the dorsal compartment containing which of the following tendons? Select one: Extensor carpi radialis longus and brevis Extensor carpi ulnaris Extensor indicis and extensor digitorum communis Extensor pollicis brevis and abductor pollicis longus Extensor pollicis longus Certainty: C=1 (Unsure: <67%) C=2 (Mid: >67%) C=3 (Quite sure: >80%) Feedback De Quervain's tenosynovitis is one of the most common differential diagnoses of the wrist pain. The tendons of the 1st dorsal compartment of the wrist (extensor pollicis brevis and abductor pollicis longus) become inflamed usually related to overuse and less commonly wrist sprain or inflammatory joint disease. The correct answer is: Extensor pollicis brevis and abductor pollicis longus You did not select a certainty. Assuming: C=1 (Unsure: <67%). Question 77 Incorrect CBM mark 0.00 Weight 1.00
  • 65. Flag question Question text You assess a 3-week-old girl in clinic and the parents explain that there is a history of urine leaking from around the umbilicus. It is associated with local inflammation of the skin. A tentative diagnosis of urachal fistula is made. You present the history to the paediatric surgical team. What do you explain to them? Select one: The urachus develops from the mesonephric duct and drains into the urogenital sinus The urachus develops from the mesonephric duct and drains to the allantoic cavity The urachus develops from the mesonephric duct and drains to the amniotic cavity The urachus develops from the urogenital sinus and drains to the allantoic cavity The urachus develops from the urogenital sinus and drains to the amniotic cavity Certainty: C=1 (Unsure: <67%) C=2 (Mid: >67%) C=3 (Quite sure: >80%) Feedback As well as local inflammation, a granuloma may also form. Gram-positive organisms, such as Staphylococcus aureus and Streptococcus pyogenes, are also commonly identified. The correct answer is: The urachus develops from the urogenital sinus and drains to the allantoic cavity You did not select a certainty. Assuming: C=1 (Unsure: <67%). Question 78 Correct CBM mark 1.00 Weight 1.00 Flag question Question text
  • 66. A 64-year-old presents with headaches and a loss of visual acuity. You examine the visual fields and document that there is loss of visual field represented by the shaded area as shown. Which one of the following is the most likely cause of the loss of visual field? Select one: Left optic nerve lesion Left optic tract lesion Pituitary tumour Right optic nerve lesion Right optic tract lesion Certainty: C=1 (Unsure: <67%) C=2 (Mid: >67%) C=3 (Quite sure: >80%) Feedback The visual field disturbance shown is a RIGHT homonymous hemianopia, identifying loss of the RIGHT half of visual field in both eyes. The Right side of the brain has visual pathways for the LEFT visual fields and vice versa. The loss of vision shown, indicates a LEFT optic tract lesion. Right optic tract lesion would give the opposite field loss; pituitary tumour would give a bilateral temporal hemianopia; optic nerve lesions would give complete loss of vision in the respective eye. The correct answer is: Left optic tract lesion You did not select a certainty. Assuming: C=1 (Unsure: <67%). Question 79 Correct CBM mark 1.00 Weight 1.00 Flag question Question text
  • 67. A 71-year-old woman is referred having attended the opticians. She has a bitemporal hemianopia. An MRI scan shows suprasellar extension of a pituitary tumour. Which one of the following is the most likely explanation for the visual field defect? Select one: Bilateral compression of the lateral geniculate bodies Compression of both optic tracts Compression of decussating nasal retinal fibres Compression of the right and left Meyer's Loops Impaired conduction of afferent impulses from the temporal half of the retinae Certainty: C=1 (Unsure: <67%) C=2 (Mid: >67%) C=3 (Quite sure: >80%) Feedback The nasal retinal fibres decussate in the compressed optic chiasm. Distractors can all cause field defects. The correct answer is: Compression of decussating nasal retinal fibres You did not select a certainty. Assuming: C=1 (Unsure: <67%). Question 80 Correct CBM mark 1.00 Weight 1.00 Flag question Question text A 35-year-old construction worker presents with pins and needles of the radial three and a half digits of her hand that wakes her up at night and is worse after using vibrating machinery. You correctly diagnose these symptoms are due to a compressive neuropathy in the carpal tunnel. Which of the following is the correct list of structures that run through the carpal tunnel? Select one: Median nerve, 4 tendons of flexor digitorum profundas, four tendons of flexor digitorum superficialis and flexor carpi radialis tendon Median nerve, 4 tendons of flexor digitorum profundus, four tendons of flexor digitorum superficialis and flexor pollicis brevis tendon
  • 68. Median nerve, 4 tendons of flexor digitorum profundas, four tendons of flexor digitorum superficialis, and flexor pollicis longus tendon Ulna nerve, 4 tendons of flexor digitorum profundas, four tendons of flexor digitorum superficialis and flexor carpi radialis tendon Ulna nerve, 4 tendons of flexor digitorum profundas, four tendons of flexor digitorum superficialis and flexor pollicis brevis tendon Certainty: C=1 (Unsure: <67%) C=2 (Mid: >67%) C=3 (Quite sure: >80%) Feedback The carpal tunnel contains the medial nerve which supplies sensation to the radial three and a half digits consistent with the symptoms stated above. There are 9 tendons that run through the carpal tunnel, these are correctly stated in option C. The correct answer is: Median nerve, 4 tendons of flexor digitorum profundas, four tendons of flexor digitorum superficialis, and flexor pollicis longus tendon You did not select a certainty. Assuming: C=1 (Unsure: <67%).